Complete Endocrinology Objectives

Pataasin ang iyong marka sa homework at exams ngayon gamit ang Quizwiz!

Symptoms associated with hypercalcemia

"bones, stones, abdominal groans, moans, with psychiatric overtones" and fatigue

Types of *Hyperparathyroidism*

*Primary* (PHPT)Tumor of the parathyroid gland causes too much PTH even though calcium is high *85% single adenoma* (the vast majority) 5% hyperplasia <1% carcinoma Can be associated with MEN 1 or MEN 2 *Secondary* (Ca is low, so body makes more PTH) Results from *hypocalcemia* and defeciency in vitamin D Normal physiologic response Any cause of hypocalcemia can cause this *Tertiary* (usually stage 4 kidney failure) The parathyroid glands have sensed so much lack of calcium for so long due to kidney failure and the lack of D3 that they become "Autonomous" enlarged, and constantly pump out PTH PTH levels extremely high

The Gonads (ovaries and testies)

*Produce germ cells* (Ova and spermatazoa) Synthesize *sex hormones* for both female and male reproductive organs, secondary sex charachteristics *There are three types of sex hormones* Primarily women have Estrogen Progestogens Primarily men have Androgens

Prior to parathyroidectomy, how do you locate the adenoma

*Sestamibi scintigraphy* - the one that stays dark is the one you're removing. Other imaging can help: US neck 4D - CT neck MRI PET-CT

Describe the pathophysiology, epidemiology, risk factors, clinical presentation, diagnostic evaluation, and management of: *Myxedema crisis (coma)*

*Severe, life threatening hypothyroidism* WHY: Usually the result of long-term, undiagnosed hypothyroidism. A myxedema coma may be triggered by sedatives, infection or other stress on the body. PRESENTATION: Usual symptoms & signs of hypothyroidism, PLUS -*Hypothermia (80 % of cases)* -*Hypotension / bradycardia* -*Hypoventilation / respiratory failure Ileus* -*Depressed mental status / coma* -Low voltage EKG with prolonged QT interval -Pericardial effusion TREATMENT: -*IV Hydrocortisone FIRST, then IV Thyroxine (T4)* -Search for and treat precipitating cause -Follow TSH and FT4 levels. TSH usually decreases in 24-72 hours and normalizes later.

Define the following term(s): *Thyroiditis*

*Swelling (inflamation) of the thyroid gland*, sometimes causing pressure symptoms in acute and subacute forms; painless enlargement and rubbery firmness in chronic forms MULTIPLE CLASSIFICATIONS (SEE NEXT CARDS) -*Hashimoto's thyroiditis:* *most common thyroid disorder in the United States* -Subacute thyroiditis -Infectious (suppurative) thyroiditis -Postpartum thyroiditis

Describe the actions and feedback mechanisms of thyroxine (T4), triiodothyronine (T3), and thyroid-stimulating-hormone (TSH).

*T4 (Thyroxine)- inactive hormone*: -The majority of T4 is converted to T3 within the peripheral tissue. *T3 (Triiodothyronine)—active hormone*: -Thyroid hormones (specifically T3) regulate rate of overall body metabolism -*T3 increases basal metabolic rate* -T3 increases oxygen consumption by most peripheral tissues -*Increases body heat production* FEEDBACK LOOPS EXPLAINED (SEE PICTURE) *The positive feedback loop:* 1. TRH is secreted by the hypothalamus 2. TRH stimulates TSH secretion by the pituitary gland 3. TSH stimulates TH secretion by the thyroid gland ---->TH = T3 (20%) + T4 (80%) *The negative feedback loop:* 1. T3 reduces TSH secretion 2. T4 reduces TRH secretion NOTE: Without *IODINE* TH cannot be synthesized from precursors!

Define the following term(s): *Thyrotoxicosis*

*The clinical syndrome of hypermetabolism resulting from increased free thyroxine (T4) and/or free triiodothyronine (T3) serum levels* -Thyrotoxicosis can result from the destruction of thyroid follicles and thyrocytes in the various forms of thyroiditis, it can be caused by an excessive intake of exogenous thyroid hormone, or excessive production. -The term thyrotoxicosis is *not synonymous with hyperthyroidism, but it does include it!* -*The most common form of thyrotoxicosis is Graves disease* (Graves is explained on another card) UNDERSTAND THE DIFFERENCE (AKA too long; didn't read) -*Hyperthyroidism is the condition that occurs due to excessive production of thyroid hormone by the thyroid gland* -*Thyrotoxicosis is the condition that occurs due to excessive thyroid hormone of any cause and therefore includes hyperthyroidism*

Define the following term(s): *Euthyroid*

*The state of having a normally functioning thyroid gland*

Posterior pituitary in detail

*Vasopressin* AKA *ADH*- Released with low blood volume and low Na. Makes the kidneys re absorb water and Na (V2 receptors), makes blood vessels constrict (V1 receptors). High BP and Alcohol inhibit it *Oxytocin* -Stimulates uterine contraction in childbirth (positive feedback keeps the posterior pituitary producing this, and causing contractions until the baby is born) and causes milk ejection in response to suckling. Also Jim says it's the love hormone. helps you love your child after its born. Also boosts sexual arousal and is present during an orgasm... doubt the whole orgasm thing will be on the test but hey, it's a fun fact.. Thanks Jim.

Define the following diseases/disorders to include the pathophysiology, epidemiology, risk factors (if any), clinical presentation, physical findings, diagnostic evaluation, differential diagnoses, and management plan: • *Hirsutism*

- Hirsutism = cosmetically unacceptable terminal hair growth that appears in women in a male pattern Causes: - Idiopahtic or familial - Polycystic ovary syndrome (PCOS) - Ovarian hypthecosis - Steroidogenic enzyme defects Symptoms and Signs - Increase sexual hair (chin, upper lip, abdomen, and chest) - Acne - Menstrual irregularities or amenorrhea Diagnosis: - Serum Total and free testosterone (Total > 200 ng/dL or free > 40 ng/dL) - Pelvic Examination and Ultrasound - Adrenal CT scan Treatment: Symptoms relief: - Shaving / Waxing - Laser hair removal - Medications (Spironolactone, Flutamide) *Treat the cause* - Tumor = Surgery

Understand how glycosylated hemoglobin (A1C) is measured

-A1c is a measure of average blood glucose over 2-3 month span (RBC life) -Glucose can bind to hemoglobin (glycated hemoglobin) -Thus the amount of hemoglobin bound with glucose is called your hemoglobin A1c!!!!! -The percentage given in an A1c is just that the percentage of hemoglobin bound by glucose -The more glucose in your blood the more is likely to be bound to hemoglobin -Changes in the past month are the most influential on A1c though so improved glucose control can show even after 1 month on A1c

Identify and summarize the functions of the hormones synthesized in each anatomical region of the adrenal gland.

Adrenal Gland has 2 regions: 1) Adrenal Cortex: has 3 layers: - Zona glomerulosa (outer) → aldosterone (mineralocorticoid) ○ Aldosterone: reabsorb Na, excrete potassium, retain water and increase BP *Stimulated by Ang II and volume status, NOT the pituitary/hypothalamus* - Zona fasciculata (middle) → cortisol (glucocorticoid) ○ Cortisol: Mediates the stress response, regulates protein, lipid and CHO metabolism, catabolic effects on bone, lymph, CT, fat and skin, fetal lung development, blood pressure, regulate GFR - Stimulated by ACTH *Cortisol is responsible for the negative feedback on the Hypothalamus and pituitary* - Zona reticularis (inner) → adrenal androgens (testosterone, estradiol) ○ Testosterone: develops male reproductive tissue (testes and prostate), and secondary sex characteristics ○ Estradiol: required for reproduction and sexual function - Stimulated by ACTH 2) Adrenal Medulla---> - Catecholamines (Epi/Norepine) - Metabolites (Metanephrines/Normetanephrines)

Understand the differences between the insulin preparations (bioavailability) and appropriateness of use

All of these insulins must be given sub cutaneously

When to treat hyperclacemia

Asymptomatic or mildly symptomatic doesnt require tx if *Ca<12 mg/dl* does not require immediate treatment *Ca< 12-14 mg/dL* may be tolerated chronically and may not require immediate therapy To treat either of these: *Avoid ca > 1000 mg/day, dehydration, aggravating meds and prolonged inactivity* *Calcium >14 mg/dL* requires therapy, regardless of symptoms

Differentiate the major causes hypercalcemia

Causes of hypercalcemia can be seperated into *PTH dependent* and *PTH Independent* *PTH dependent* Primary Hyperparathyroidism -*adenoma* -4 gland hyperplasia (Enlargement of all four parathyroid glands) Familial hypocalciuric hypocalcemia Tertiary Hyperparathyrioidism *PTH independent* Malignancy -Direct bone invasion -PTHrp release -Increased 1,25 vit D Vitamin D intoxication Medication (thiazides, Lithium) Granulomatous disease Hyperthyroidism Vitamin A intoxication

Based on a clinical scenario determine which patients should be screened for diabetes mellitus

Checking by ordering a HbA1c Check everyone at 45 years old and repeat every 3 years Check at 30 or younger if: -BMI > 25 or central obesity -Sedentary lifestyle -First degree relative with diabetes -member of high risk ethnicity (AA, Latino, hispanic, Native american, Asian, pacific islander) -Had gestational diabetes, baby > 9 lbs, or perinatal deaths -HTN > 140/90 -HDL < 35 -Triglycerides > 250 -Previously elevated blood glucose on tests -A1c > 5.7% or issues with fasting glucose -Any signs of insulin resistance (Acanthosis Nigricans) -History of CVD/ CAD etc.

Describe the pathophysiology, epidemiology, risk factors, clinical presentation, diagnostic evaluation, and management of: *Infectious (Suppurative) thyroiditis*

Complicated and probably not on the test unless they feel like being horrible. WHAT: Also known as subacute lymphocytic thyroiditis or "Hashitoxicosis." It can occur spontaneously or be triggered by certain medications -*Nonviral infection of the thyroid gland* -usually bacterial -Occurs mostly in women -Most likely affects those with preexisting thyroid disease, or immunosuppressed PRESENTATION: -*High fever* -*Redness of overlying skin* -Thyroid gland tenderness -Cervical lymphadenopathy DIAGNOSIS: -Labs show variable abnormalities or may be euthyroid (not helpful) -Thyroid autoantibodies (if obtained) are absent -*C-Reactive Protein (C-RP) or Erythrocyte Sedimentation Rate (ESR) increased* -*Thyroid Fine Needle Aspirate (Thyroid FNA)* --->Send for gram stain and culture -CT of the neck with contrast can be used TREATMENT: -*Intensive antibiotic treatment* -Occasional incision and drainage are required

Define and discuss the following terms: Homeostasis

Constant internal environment as maintained by physiological processes In many hormonal processes this occurs in the form of a negative feedback

Define the following diseases/disorders to include the pathophysiology, epidemiology, risk factors (if any), clinical presentation, physical findings, diagnostic evaluation, differential diagnoses, and management plan: *Virilization*

Its like Hirsutism but worse! - Virilization = the development of male physical characteristics, such as pronounced muscle development, deep voice, male pattern baldness. Cause: - Androgen-producing neoplasm Symptoms and signs: - Hirsutism symptoms - Frontal balding - Muscularity - Deepening voice - Clitoromegaly Diagnosis: - Serum Total and free testosterone (Total > 200 ng/dL or free > 40 ng/dL) - Pelvic Examination and Ultrasound - Adrenal CT scan Treatment: Symptoms relief: - Shaving / Waxing - Laser hair removal - Medications (Spironolactone, Flutamide) *Treat the cause* - Tumor = Surgery

Pancreas hormones (for real though)

Just kidding. * Glucogon and Insulin* from the *ilsets of langerhans* are crazy important because they regulate glucose Increased blood glucose stimulates insulin secretion and decreases glucagon secretion *Insulin is anaboloc, allows you to form carbohydrates, proteins and lipids, while Glucogon is catabolic and will tear them down and increase the amount of their base components in the blood*

Hyperparathyroid Sx

Leads to *hypercalcemia* *Loss of bone density, PTH takes calcium from the bones and puts it in the blood* Neuromuscular- paresthesias, muscle cramps and weakness, and diminished deep tendon reflexes. CNS- Malaise, headache, fatigue, intellectual weariness, insomnia, irritability, and depression. Cardiovascular- hypertension, palpitations, bradyarrhythmias, heart block, asystole Renal - polyuria and polydipsia, renal calculi

What types of cells produce testosterone? (not an objective)

Ledig Cells (interstitial cells) -->Stiumlated by LH

Sx of hypocalcemia and When to treat hypocalcemia

Neuromuscular irritability Paresthesias Chvosteks' sign (facial twitch when tapped) Trousseau's sign (tensed hand) Prolonged QT interval Broncho, carpal or laryngeal spasm Seizures treat if *Symptomatic* Or if asymptomatic with serum calcium below *7.5, a Hx of a Sz, or an abnormal EKG*

Pancreas hormones

Not important.

Parathyroid hormones

Parathyroid glands are four pea sized bodies located behind the thyroid gland that produce *PTH*. PTH is stimulated by *low blood calcium* Many of the functions of PTH require 1,25-dihydrocholecalciferol which comes from *vitamin D* It *increases calcium levels in the blood* by increasing calcium reabsorbtion and excreting phosphate by the kidneys (phosphate binds up calcium, so less means more free calcium). Also increases resorption of calcium from bone to blood. Also makes the intestines absorb more calcium

Define the following diseases/disorders to include the pathophysiology, epidemiology, risk factors (if any), clinical presentation, physical findings, diagnostic evaluation, differential diagnoses, and management plan. • Prolactinomas

Prolactin secreting pituitary adenoma Causes: - Sporadic - MEN type 1 or 4 Clinical features: - amenorrhea - menstrual irregularities - infertility - galactorrhea (women >> men) - impotence - loss of libido (men) - mass effect symptoms (visual loss, headache) Labs: Evaluate for - Pregnancy (hCG) - Hypothyroidism (FT4 & TSH) - Kidney disease (BUN & creatinine) - Cirrhoisis (liver tests) - Hyperparathyroidism (serum Ca) - Hypogonadism (Testosterone/estrodiol, LH, and FSH) Check: - Prolactin level (The larger the tumor, the higher the prolactin level) Diagnosis - MRI of pituitary Treatment = First Line: - *dopamine agonists (bromocriptine, cabergoline)* If patient cannot tolerate drugs or tumor is not responsive, then: Second Line: - Transsphenoidal surgery with radiation therapy)

Explain the significance of and consequences of proteinuria in the diabetic patient

Proteinuria is a sign of nephropathy happening. This is bad and will lead to ESRD and glomerulosclerosis Need to watch kidney function in diabetics and prescribe ACE-I if proteinuria is found.

Define the following disease: Pathophysiology, Epidemiology, Risk factors, Clinical presentation, physical findings, diagnostic evaluation, Ddx, management for *DKA (Diabetic Ketoacidosis)*

REALLY BAD OUTCOME FOR DIABETICS -More common in type 1 -Blood glucose above 250 -Can happen over hours or days depends on how high and whats going on... -Ketosis (ketones are a biproduct of lipid metabolism) -*pH less than 7.3 or HC03 less than 15 mEq/L* -Anion gap metabolic acidosis *Symptoms:* -*dehydration* -*N/V* -*Excessive thirst* -polyuria -weakness -Fatigue -Anorexia -Tachycarida -*Hyperventilation* -Keytones -Coma *Causes:* -*new onset diabetes* -infection -*problems with insulin administration* -use of prednisone or stress *Diagnosis* -*Check plasma glucose level* -ABG (check pH) -Check BMP (for anion gap) -Urine keytones Treatment: -INSULIN -IV fluids 0.9% NaCl -Closely monitor vitals and labs until back into normal level

Define and discuss the actions of releasing and target hormones

Releasing - induce production and secretion of hormones from other glands (ex- Hormones released from the hypothalamus to the pituitary telling it to release hormones) Target hormones - hormones that directly elicit a reaction from an organ (ex- Hormones released from the pituitary that affect the gonads directly

Testicles

Sertoli cells- babysitters that maintain blood testes barrier, and nourish sperm cells, make fluid, synthisize inhibin. (stimulated by FSH) Testicles have 200X higher levels of testosterone than the rest of the body. If you take testosterone then the testosterone in your blood inhibits testosterone synthesis in the testicles and can cause infertility. Leydig cells make testosterone (stimulated by LH) Spermatogenesis takes 72 days and sperm start as a primordial germ cell, then become spermatogonia then mitose into primary (4dna) then secondary (2n DNA) then spermatids (1dna)

Positive feedback

Target gland hormone acts back on the hypothalamus and/or pituitary to *increase* the release of hormones that stimulate the secretion of the target gland hormone. Requires a "threshhold level" at which it eventually shuts off Ex) During a menstrual cycle estrogen temporarily stimulates hormone release from pituitary and hypothalamus instead of inhibiting it

Describe the main actions of PTH and vitamin D in the maintenance of calcium and phosphorus homeostasis

The bones act as the calcium reservoir. Plasma calcium is used by the nerves/muscles is only a small fraction. *normal serum calcium is 8.5-10.4* The parathyroid gland senses that you have low Ca, it releases PTH to the *Kidneys and the osteoclasts in the bones*. The osteoclasts are release the calcium *quickly* for urgent use, the kidney's *reabsorb calcium and excrete phosphate*. Phosphate binds up calcium so PTH gets it out of your system. PTH also make the kidneys activate vitamin D3 into 1,25 (OH)2D3 AKA *Calcitrol*, and this "active" vitamin D *increases absorbtion of calcium and phosphate in the intestines* Parathyroid glands have *Ca sensing receptors* (CaSR) for negative feedback

Adrenal medulla hormones

The catecholamines *adrenaline and noradrenaline AKA Epi/Norepi* The fight or flight hormones Cause liver to increase gluconeogenesis and glycogenolysis for energy Inhibits glucose uptake into adipose, and casues fat breakdown for energy Decrease glucose uptake into muscles *Decreses insulin, increases glucogon* Epi is made from nor epi (L-tyrosine-Ldopa-Dopamine-NEpi-EPI) The adrenal gland is the only effector organ of the sympathetic nervous system attached directly to a postganglionic fiber. The catecholamines are produced by *chromaffin glands*, and acytylcholine is the neurotransmitter that triggers their release (tumor in these glands= pheochromocytoma=increased Epi/norepi secretion) 90% of chromaffin cells secrete epi, other 10 secrete norepi Epi and norepi aree made from tyrosine, and cause negative feedback by inhibiting tyrosine hydroxylase when there is too much. t1/2 of 1-3 minutes before COMT and MAO destroy them in the liver and kidneys

Explain the usefulness of suppression and provocative testing and when timing of testing is indicated with adrenal gland disorders.

The goal of these tests is to identify what step in the process is malfunctioning - If overexpressing, do an inhibitory test - If insufficient, do a stimulatory test Testing should be done in the morning! (Peak cortisol levels occur between 6 to 8am)

Posterior Pituitary

The posterior pituitary *does not produce its own hormones*; instead, it stores two hormones made in nucleuses up in the hypothalamus -*vasopressin* Made in the supraoptic nucleus or *SON* -*oxytocin* made by the paraventricular nucleus or *PVN*

Thyroid hormone *calcitonin*

The thyroid also produces *calcitonin* in the *parafollicular cells* which reduces calcium release from bones into blood, and inhibits bone erosion and absorbtion of calcium into the kidneys. (opposite of PTH)

Describe the clinical use and contraindications of corticosteroids and list examples of common glucocorticoids and mineralocorticoids.

Cortisol is a steroid hormone that is normally secreted by the adrenal cortex in response to ACTH. It exerts its action by binding to nuclear receptors, which then act upon chromatin to regulate gene expression, producing effects throughout the body. - Corticosteroids are used to replecate the same action as Cortisol. Potential side effects which include: - insomnia - cognitive and personality changes, - weight gain with central obesity - bruising, striae - muscle weakness - polyuria - sex hormone suppression - candidiasis - opportunistic infections - Hyperglycemia (in diabetics) At high doses = cardiovascular effects, (dyslipidemia, myocardial infarction, stroke, atrial fibrillation or flutter, and heart failure) Prolonged use = suppressed pituitary ACTH secretion = secondary adrenal insufficiency Common glucocorticoids: hydrocortisone, prednisone/prednisolone, dexamethasone, betamethasone, fluticasone Common mineralocorticoid: fludrocortisone (Synthetic Aldosterone)

Case #1 - 70 year old man with Bp= 190/100 - Normal physical exam - BMI=24 - K= 3.4 - Aldo= 40; renin= <0.6 - Salt load: 24 hr urine= Na>200, aldo=17mcg/24hrs A CT is done and a small adenoma is identified in the left adrenal. Should we send this patient to? A. Surgeon B. Start medical therapy C. Watch and wait D. Adrenal venous sampling (AVS)

D. Adrenal venous sampling (AVS) The patient should be sent for Adrenal venous sampling (AVS) for aldosterone levels This is to identify which adrenal is for sure the culprit for the hyperaldosteronism

Case #4 - 52 year old patient is found to have a left incidental adrenal mass of 2.5cm when being evaluated in ER for abdominal pain. - No medications - Normal blood pressure - Normal CMP and CBC What's next? A. Biopsy this mass B. Check a potassium level C. Consult medical oncology D. Check plasma metanephrines

D. Check plasma metanephrines

Discuss nonpharmacologic management and patient education of hyperglycemic states as it relates to diet and exercise

Diabetic education is very important. there are diabetic educators that get paid lots of money to tell diabetics how to manage their life. Exercise is very important (30 min cardio daily, resistance training 2x week) Diet (especially in T2DM) is crucial and can even cure a diabetic patient but this rarely happens.

Define the following diseases/disorders to include the pathophysiology, epidemiology, risk factors (if any), clinical presentation, physical findings, diagnostic evaluation, differential diagnoses, and management plan. *Vitamin D deficiency*

Due to decrease in skin production, obeisity, poor nutrition, or use of phenytoin (increases Vit. D metabolsim), secondary hyperparathyroidism Sx: Bone pain, muscle weakness, Fx, osteomalacia, waddling gait, spasms, cramps, Sx of hypocalcemi Labs- *25 OH D <15-20*, *elevated PTH* , Elevated Alk Phos, low calcium, low urinary calcium Tx: *Vitamin D replacement*, goal serum vitamin d of 30-50

Define the following diseases/disorders to include the pathophysiology, epidemiology, risk factors (if any), clinical presentation, physical findings, diagnostic evaluation, differential diagnoses, and management plan. • Dwarfism

Dwarfinsm *Caused by decreased secretion of GH* Typical symptoms: - Delayed growth and development Treatment: - GH replacement *(May not need to know this but it is the only thing specifically about Dwarfism in the book)* Dwarfism caused by Laron syndrome - An autosomal recessive disorder that is mainly caused by *mutations in the gene* that encodes the GH receptor, resulting in *GH-resistance* - This causes a severe deficiency in serum insulin-like growth factor-1 (IGF-1). Clinical Presentation: - prominent forehead - depressed nasal bridge - small mandible - central obesity. - Possible hypoglycemic seizures

Case #2 - 65 year old woman with a 2 day history of severe muscle weakness. - HPI: significant weight gain and type 2 diabetes over last 2 years - Meds: Lisinopril, amlodipine, metoprolol, metformin - Vitals: Bp=154/92, BMI=40 Labs: - Creatinine: 1.3mg/dL - Glucose: 145 mg/dL - K: 2.9 meq/L What tests will help to identify the etiology of her symptoms? A. Adrenal CT B. Mid afternoon aldosterone level C. Pituitary MRI D. Pancreatic MRI E. 24 hr urine for free cortisol

E. 24 hr urine for free cortisol

Discuss the following medical complications to include treatment options associated with diabetes: *Disorders of the foot*

Foot Care is VERY important for diabetic patients -Need to prevent ulcers and amputations -Educate the patient on foot care, wearing compression socks, reducing body weight, how to clip nails, wound care, -*MONOfilament is the most sensitive test for diabetic neuropathy (will be on the test)* -Refer to podiatrist when needed

Discuss screening and treatment decisions in people with osteoporosis based on the FRAX algorithm and the US Preventative Services Task Force recommendation.

Frax score is an algorithm developed by the World Health Organization to better predict an individual's 10-year risk of hip or other major osteoporotic fracture. You would go to their website to enter in the algorithm. *when to screen: women > 65, and women < 65 with FRAX score > 9.3%.* Evidence in men insufficient You screen with DXA (AKA DEXA)

Famelial hypocaliuric hypercalcemia (not an objective)

Genetic problem in which the parathyroid glands falsely "sense" hypocalcemia and inappropriately release slightly excessive amounts of PTH. The renal tubule calcium sensing receptors are also affected, causing hypocalciuria. Rule this out when establishing Dx of PHPT

Outline the feedback mechanisms of common endocrine systems *HPT axis*

How your body controls your metabolism You need to generate some energy or raise your body temp. 1)Hypothalamus secretes Thyrotropin releasing hormone *TRH* 2)TRH causes pituitary release of thyroid stimulating hormone*TSH*, 3)Thyroid produces and releases T4 and T3 (which also act as a negative feedback)

Outline the feedback mechanisms of common endocrine systems *HPA*

How your body reacts to stress 1) Hypothalamus releases Corticotropin releasing hormone *CRH* as a reaction to stress, the sleep wake cycle, or due to neurotransmitters. 2)CRH stimulates anterior pituitary to produce Adrenocorticotropic hormone *ACTH* (too much ACTH= Cushings) 3) ACTH activates *adrenal* hormone production primarily *cortisol*, which protects the body from stress and mediates specific physiological effects of this hormone system ( unresponsive adrenal cortex= Addisons disease) 4) (negative Feedback)- cortisol inhibits the production of CRH and ACTH

*glucogon*

Increases blood glucose levels. Produced in the alpha cells of the islet of langerhans. Opposite effects of insulin, increases glycogen breakdown and gluconeogenesis. Opposites of insulin stimulation stimulate glucogon Increases blood glucose, fatty acids, and keto acids

*Tanner stages of development* of secondary sexual characteristics Accurately assign the correct Tanner stage based on physical findings -*Pubic hair for Boys and Girls*

YOU CAN STAGE IN OTHER WAYS, BUT FOCUS ON THIS WAY FOR THE TEST -Wow this crap is convoluted *Development of Pubic Hair* Stage 1: *Prepubertal* (Think vellus hair or like NO hair) Stage 2: *Sparse growth of long, STRAIGHT (for the purpose of this exam) slightly pigmented hair*, at base of penis or along labia Stage 3: *Darker, coarser and more curled hair, spreading sparsely* over junction of pubis Stage 4: *Hair adult in type, but covering smaller area than in adult*; *no spread to medial surface of thighs* Stage 5: *Adult in type and quantity, with horizontal distribution*

Hyperparathyroidism Tx

You will either monitor it, or conduct a parathyroidectomy. If asymptomatic then *monitor* Yearly serum Ca++ Yearly Plasma Cr BMD scan every 1-2 years Surgery is indicated if they are *symptomatic* * h/o fragility fracture or clinical h/o renal stones automatically qualifies for surgery* Also qualify for sugery if. Plasma calcium >11.4 eGFR <60 ml/min (kidney failure) BMD: T-score <-2.5 at any site (osteoporosis) Prior *asymptomatic vertebral fracture* by XR/CT/MRI/VFA 24 hour urinary ca >400 mg/day (they are dumpinga ton of of excess calcium) Age <50 years Presence of nephrolithiasis/nephrocalcinosis on XR/USG/CT

Define the following diseases/disorders to include the pathophysiology, epidemiology, risk factors (if any), clinical presentation, physical findings, diagnostic evaluation, differential diagnoses, and management plan. *Hypoparathyroidism*

Your Parathyroid gland is damaged, or doesn't work due to: Post thyroidectomy, autoimmune, congential, familial, (hemochromatosis, sarciodosis) Dx: Hypocalcemia indicated by Sx and labs Low PTH labs tx: First choice- give Calcium supplaments, and Vitamin D (calcitrol or ergociferol) If that doesnt work over time then Give SubQ PTH

Discuss the following medical complications to include treatment options associated with diabetes: *Retinopathy*

Your eyes get destroyed remember. The pericytes are damaged due to glucose being toxic, then the blood vessels die (non-proliferative) then new ones form (proliferative) -95-100% of T1DM have this after 20 years of diagnosis -60-80% of T2DM have this after 20 years of diagnosis Causes: -Poor glycemic control -Most common cause of blindness in the world *Management* -Dilated fundoscopic exam yearly by ophthalmologist -Manage blood glucose well and hit A1c goal of 7% or less

Define the following diseases/disorders to include the pathophysiology, epidemiology, risk factors (if any), clinical presentation, physical findings, diagnostic evaluation, differential diagnoses, and management plan. *Osteomalacia*

bone matrix *intact*, mineral decreased (Osteoporosis and osteomalacia often coexist in the same patient.) Results from *inadequate calcium, phosphate or vitamin D* Most often vitamin D When growth plates are open (AS IN CHILDREN) it causes *Rickets* Sx:Bone pain Deformity Fracture Proximal myopathy Hypocalcemia (vit D def) Rickets Sx: Growth retardation Bone pain, fractures in unusual locals (scapula, pubic rami) *Skeletal deformity* -*Bowing of long bones* -Widening of the growth plates Labs: Reduced vitamin D, Hypophosphatemia, low calcium, High PTH Give Tx: Give Vit. D (ergocalciferol or cholecalciferol), Calcium, and phosphate and treat any underlying cause

Define the following diseases/disorders to include the pathophysiology, epidemiology, risk factors (if any), clinical presentation, physical findings, diagnostic evaluation, differential diagnoses, and management plan. *Osteoporosis*

bone matrix and mineral *both* decreased. Seen in women more than men This is low bone mass, disruption and skeletal fragility due to hypogonadism(men) , aging, menaupause (women) There is increased Fracture propensity of spine, hip, pelvis, and wrist from demineralization. Xrays usually show deterioration Labs: Serum PTH, calcium, phosphorus, and alkaline phosphatase usually normal. Serum 1,25-oh vitamin D levels often low Treatment: Bisphophonates are first line, and prevent calcium resorption into the blood

Discuss the difference between Cushing's disease and Cushing's syndrome.

o Cushing's disease: Pituitary tumor causing of high ACTH = Hypercortisolism o Cushing's syndrome: Adrenal is hyper-secreting cortisol due to many causes

Progestogens

primary is *progesterone* Made by the ovaries, this is what *causes changes in the uterine lining* in preperation for pregnancy, and helps out the estrogens to prepare for lactation

Estrogens

principle one is *estradiol*- exert feminizing effects. Make the breasts, uterus, and vagina mature in puberty. changes fat patterns so that women look like women. Contribute to lactation, libido, and menstrual cycle regulation. Main site of porduction is in the ovaries. Estrogen production in ovaries ceases in menopause and can lead too hotflashes, palpitations, anxiety, depression, and osteoperosis. In men a little bit from the adrenal glands is needed to for bone remodeling and closure of epiphyseal plates

The adrenal glands are

small structures located on top of the kidneys. Structurally, they consist of an outer layer (i.e., the cortex) and an inner layer (i.e., the medulla). *adrenal cortex* produces numerous hormones, primarily corticosteroids (i.e., glucocorticoids and mineralocorticoids *The adrenal medulla* — adrenaline and noradrenaline AKA Epi/Norepi The adrenal glands also make just a liiiiitle bit of the sex hormones made by the gonads

Define and discuss the following terms: Negative feedback

when hormones released by the target gland reach a certain predetermined level in the blood, the previous gland ceases hormone release. Can also be "short loop feedback" in which the hormone shuts off the single gland that made it. At different stages of life the amount of hormone required to initiate feedback can change.

Define the following term(s): *Goiter*

*A goiter is an abnormal enlargement of your thyroid gland* -usually painless -can cause a cough and make it difficult to swallow or breathe if large enough THE MOST COMMON CAUSES -Worldwide: *Lack of iodine in the diet* -United States: We use iodized salt here in the USA. Our most common causes are due to *over/underproduction of thyroid hormones* or *nodules that develop in the gland itself*

list and discuss the importance of goal values for A1C, hyperlipidemia, and BP for a diabetic

*A1c goal*: -less than 6.5% is ideal but -*Less than 7% is good* *Hyperlipidemia goals:* LDL < 100 Triglycerides < 150 HDL > 40 *BP goals:* -Less than 130/90

Anterior Pituitary hormones in detail

*ACTH, gonadotropins (LH and FSH), and TSH* act on other glands *GH and Prolactin* act on target organs *adrenocorticotropic hormone (ACTH)*- stimulates the adrenal cortex to produce corticosteroid hormones— primarily *cortisol* and some sex hormones *gonado tropins (FSH and LH)*- FSH stimulates follicles in ladies to start up cycle, stimulates sertoli cells in men. LH surgevstarts mentruation in women, activates leydig cells in men *TSH* Stimulates the thyroid gland to release thyroid hormone *GH- Growth hormone* *Most abundant*- Makes your body grow... (bones, fat, muscle, sexual organ development etc.) Higher as a child. Also *regulates carb, protein and fat metabolism* and *decreases glucose absorbtion, and increases gluconeogenesis* Increases muscle mass, decreases adiposity it does this by either acting on organs directly, or acting on the *liver and kidneys which makes (insulin like Growth factor-1) IGF-1* which has a longer half life than normal GF It also tells the liver make *somatomedins* that make you grow (acromegaly- Excess GH in adulthood, Gigantism- Excess GH before puberty) *Prolactin*- makes breasts in ladies (woohoo) and makes them produce milk after birth. Reduces male sex drive and leads to impotence (lame...)

Describe the screening laboratory methods used to evaluate patients with excess as well as deficiencies in adrenal hormones.

*Adrenal insufficiency / Addison's disease* - AM cortisol (between 6 - 8 am): ○ >19 = intact HPA axis ○ <5 = definite adrenal insufficiency ○ 6 - 18 = indeterminate - Cosyntropin (ACTH) stimulation test: ○ Synthetic form of ACTH. Injected and measure serum cortisol levels at 0, 30, and 60 minutes ○ Cortisol level >19 at any point indicates adequate adrenal gland activity *Cushing's Syndrome:* - 24 hour urine free cortisol - 1 mg overnight dexamethasone suppression test = 1st - 1 mg Dexamethasone is taken at 11PM 2nd - 8 AM serum cortisol level is drawn the next morning (plasma cortisol value less than 1.8 mg/dL is normal) To identify and locate the cause: 1st: *Check plasma ACTH level* - Low / Suppressed (<10) = adrenal source - Normal or high (20-100) = Cushing's disease - Very high ACTH (>100) = ectopic source *Primary (hyper)aldosteronism* - 8 am plasma aldosterone (PA) and plasma renin (PRA) levels. Obtain PA/PRA ratio. Plasma Aldosterone > 15 ng/dL Ratio (Ald/Renin) > 20 (4 -10 is normal) Confirmatatory testing: Saline or salt loading test with 24-hour urine aldosterone (If salt fails to suppress aldosterone = Hyperaldosteronism) To locate cause: - Adrenal CT then, if needed - Adrenal venous sampling (AVS) for aldosterone levels (This is to identify which adrenal is FOR SURE the culprit for the hyperaldosteronism) *Hirsutism* & *Virilization* - Serum Total and free testosterone (Total > 200 ng/dL or free > 40 ng/dL) *Pheochromocytoma* Urinary measurements of: - 24 hour urine free catecholamines (dopamine, epinephrine, norepinephrine) - Elevated Plasma metanephrines

Type II diabetes drugs

*Metformin* is the Type II GOD drug!!!! MOA: Hepatic insulin sensitizer -NO Hypoglycemia -not metabolized in liver -Always improves hyperglycemia -Improves hyperglycemia, insulin resistance, triglycerides, cholesterol, NO WEIGHT GAIN, *Sulfonylurea* -Used in combo with metformin *Thiazolidinendiones* -increases insulin sensitivity -decreases FFA -Decreases adipocytokines -improves lipid storage -Can be used in pre diabetics -Weight gain, water retention, bone loss :( -Improves fatty liver disease *These drugs enhance insulin secretion:* -Sulfonylureas -Dpp-IV inhibitors -GLP-1 agonist (incretin mimetics)

Describe the pathophysiology, epidemiology, risk factors, clinical presentation, diagnostic evaluation, and management plan of: *Amenorrha*

*Amenorrhea = Absence of Menses in presence of normal growth and secondary sex characteristics* -Estradiol will be low in BOTH. To differentiate, look at the patients FSH, and LH! -------------THERE ARE TWO TYPES------------ *Primary: no menses in female > 15 yrs* *PRIMARY = DEFECT/DAMAGE IN THE OVARIES* -------------CAUSES OF PRIMARY----------------- - gonadal dysgenesis - *ovarian failure* (primary ovarian insufficiency) - *mullerian agenesis (absence of vagina)* - kallmans syndrome - Androgen insensitivity syndrome (resistant to T) and 5-alpha reductase defieicney (ambiguous genitalia) -*Turners syndrome* DIAGNOSIS (SEE PICTURE): -Thorough history and physical examination -*Labs to check serum levels of* --->*FSH (follicle stimulating hormone) (HIGH)* --->*LH (luteinizing hormone) (HIGH)* --->*Estradiol (LOW)* TREATMENT: -*Treatment of primary amenorrhea is directed at the underlying cause* -Girls with permanent hypogonadism are treated with hormone replacement therapy -*Birth control* (estrogen and progesterone replacement) ------------------------------------------------------ *Secondary*: absence of menses for > 6 months in females post-menarche (the first occurrence of menstruation). ------------CAUSES OF SECONDARY-------------- - *pregnancy* - *functional hypothalamic amenorrhea: (exercise, eating, stress. Results in decreased GnRH)* - *Polycystic ovarian syndrome* - hyperprolactinemia - thyroid dz - pituitary tumor - hyperandrogenism - autoimmune dz - ashermans syndrome (uterine scaring) - Chemotherapy DIAGNOSIS: -Thorough history and physical examination -*Labs to check serum levels of* --->*FSH (follicle stimulating hormone) (LOW)* --->*LH (luteinizing hormone) (LOW)* --->*Estradiol (LOW)* TREATMENT (same as Primary): -*Treatment of primary amenorrhea is directed at the underlying cause* -Girls with permanent hypogonadism are treated with hormone replacement therapy -*Birth control* (estrogen and progesterone replacement)

Define the following diseases/disorders to include the pathophysiology, epidemiology, risk factors (if any), clinical presentation, physical findings, diagnostic evaluation, differential diagnoses, and management plan. • Gigantism

*Caused by GH hypersecretion in childhood (Before the long bone epiphyses are fused) GH stimulates the release of IGF-1 for liver and other tissue *Almost always caused by a GH secreting pituitary adenoma*

Define the following diseases/disorders to include the pathophysiology, epidemiology, risk factors (if any), clinical presentation, physical findings, diagnostic evaluation, differential diagnoses, and management plan. • Diabetes insipidus

*Central Diabetes insipidus* Caused by low ADH secretion by posterior pituitary 2 types: Primary central DI: - Low ADH secretion from the pituitary without a tumor or injury to the gland. Most often genetic Causes: - Genetic - Idiopathic Secondary central DI: - Low ADH secretion WITH tumor, infarction, or injury to the pituitary Causes: - Tumor - trauma - destructive processes involving the hypothalamus Clinical presentation: - Polyuria - polydipsia (intense) Diagnosis: CLINICAL - NO diagnostic lab tests - 24 hour urine volume > 2 L/24 h - Check serum/urine osmolality (Serum is high/normal, Urine is dilate) To confirm: - *Water Deprivation Test* (They will still pee a ton even though they are not drinking water) - MRI of the pituitary and hypothalamus if needed. Treatment: - *Desmopressin (DDAVP)* (Synthetic form of ADH) - Chlorpropamide - treat underlying cause OTHER TYPE OF DI *Nephrogenic DI* ADH secretion is normal but tubules cannot respond to ADH Causes: - commonly lithium use - hypercalcemia - pyelonephritis Treatment: - sodium restriction - thiazide diuretics (Don't know why, JUST DO IT)

Explain how the feedback loop controlling prolactin is different from the other feedback loops in the hypothalamic-pituitary axis.

*Control of prolactin* secretion by the hypothalamus is *unique* to that of the other anterior pituitary hormones in that under *normal circumstances is NOT released.* - Dopamine released from the hypothalamus inhibits prolactin release. - In the absence of dopamine, prolactin is able to be secreted. - Increases in plasma prolactin = increased levels of dopamine, which inhibit further release.

Hypothalamus

*Controls many bodily functions* including eating and drinking, sexual functions and behaviors, blood pressure and heart rate, body temperature maintenance, the sleep-wake cycle, and emotional states (e.g., fear, pain, anger, and pleasure) in addition to hormone stimulation, this can be stimulated by nerves in the brain, *serves as a major link between nervous and endocrine systems * *Releases its hormones into vessels connected to the pituitary gland* and promotes or inhibits pituitary gland hormone release

Adrenal cortex hormones

*Cortisol*- AKA hydrocortisone- *principle Glucocorticoid* *protects you from stress* This protection is vital for life. It's regulation helps control carbohydrate, protein, and lipid metabolism Cortisol tries to get all the fuel possible to the rest of your body to protect you from stress. *Cortisol opposes insulin* by increasing glucose in the blood, and it tells the liver to make storage form of glucose *glycogen*, it also converts the muscles into free amino acids that can be used for fuel. It converts any calories you eat into easy to process fat. Mild doses of this and other "corticosteroids" (steroids made in the adrenal cortex) are protective against inflammation. Normally it is released at different times of the day, with a peak when you wake up in the morning. If it is elevated at while you sleep it indicates cushings disease. (Elevated- Cushings, decreased- addisons) Side note: because corticosteroids antagonize insulin, they pretty much render insulin useless in any diabetic patients, and their BS will get suuuper high. *Aldosterone*- Principle *mineralocorticoid*- prevents sodium loss, constricts blood vessels, etc. etc. Part of the RAAS system, initiated via Renin/angiotensin, not by the pituitary or hypothalamus

Compare and contrast the differences between laboratory values found in DKA and hyperosmolar hyperglycemic state

*DKA*: -Plasma glucose > 250 mg/dL -*Elevated keytones* Plasma > 3mmol Urine 2+ of standard stick -Anion gap metabolic acidosis -*pH < 7.3* -HC03 < 15 mmol/L *Hyperosmolar Hyperglycemic state*: -Plasma glucose > 1000 mg/dL -*NO KEYTONES* -*NO ACIDOSIS (normal pH)* -Elevated serum osmolality (> 320 mmol/L) due to glucose -SEVERE Dehydration

Describe *Euthyroid sick syndrome*

*Dysregulation of thyrotropic feedback control where the levels of T3 and/or T4 are at unusual levels, but the thyroid gland does not appear to be dysfunctional* *Seen in patients without known thyroid disease who have a low serum T4 with a serum TSH that is not elevated* -This syndrome can be seen in patients with severe illness, caloric deprivation, or who have had major surgery. ■ Patients may have acquired transient central hypothyroidism ■ *Treatment of patients with thyroid hormone appears to be of little benefit and may be harmful*

Describe the pathophysiology, epidemiology, risk factors, clinical presentation, diagnostic evaluation, and management of: *Neoplasms (papillary, follicular, medullary, and anaplastic)*

*Firm nodule upon examination of thyroid* -95% benign -The female:male ratio is 3:1 -*REFER THIS PATIENT RIGHT AWAY* SEVERAL DIFFERENT TYPES: *Follicular (15%)*: can be benign or malignant carcinoma. *Medullary (5%)*: Malignant carcinoma derived from calcitonin producing parafollicular cells. *More malignant than papillary or follicular carcinomas* *Papillary (70%)*: malignant, local invasion and lymph node spread. *Anaplastic (5%)*: malignant, occurs in older individuals, very aggressive and rapidly causes pain, dysphagia and hoarseness. *Poor prognosis* RISK FACTORS: -History of thyroid cancer in one or more first degree relatives -*History of external beam radiation as a child/ Exposure to ionizing radiation in childhood or adolescence* ESSENTIALS OF DIAGNOSIS: -Painless swelling in region of thyroid. -Thyroid function tests usually normal. -Past history of irradiation to head and neck may be present. -*Positive thyroid needle aspiration (fine needle biopsy) required for diagnosis* TREATMENT: -*Total thyroidectomy* usually needed -Serum Thyroglobulin and neck ultrasound for follow up

Describe the pathophysiology, epidemiology, risk factors, clinical presentation, diagnostic evaluation, and management of: *Chronic (Hashimoto's) thyroiditis*

*Hashimoto thyroiditis (aka chronic lymphocytic thyroiditis) is an autoimmune condition. It is the most common thyroid disorder in the United States (we are iodine SUFFICIENT here).* -*Female > Male, genetic predisposition* PATHOPHYSIOLOGY: *B-lymphocytes invade the thyroid gland and destroy normal thyroidal architecture* -Results from abnormal T cell activation and subsequent B cell stimulation to secrete a variety of autoantibodies. ->*Tg Ab- Thyroglobulin antibody* ->*TPOAb - Thyroid peroxidase antibody* CLINICAL PRESENTATION: -*Hashimoto's disease typically progresses slowly over years* -*Diffusely enlarged, firm, and finely nodular thyroid gland* (one thyroid lobe may be asymmetrically enlarged) -Although patients may complain of neck tightness, *pain and tenderness are not usually present*. SIGNS AND SYMPTOMS: -Mainly those of an underactive thyroid gland (see previous hypothyroidism card). DIAGNOSIS (LABS + SYMPTOMS ARE USUALLY ENOUGH): Blood labs show: -*Tg Ab* elevated early in disease (but disappears with time) -*TPO Ab* is present early and remains -*Elevated TSH* -*Low levels of T4 (thyroxine)* Imaging: -*Ultrasound* typically shows a gland with characteristic diffuse/various densities and hypoechogenicity (decreased echo response from US) TREATMENT: -Daily use of the synthetic thyroid hormone *T4 - Levothyroxine (Levoxyl, Synthroid, etc)*

Define and discuss the following terms: Hormone, Prohormone

*Hormone* - *molecules produced by endocrine glands in response to a stimulus and released into the blood stream to target cells* Generally speaking, hormones control the growth, development, and metabolism of the body; the electrolyte composition of bodily fluids, and reproduction Target cells have "receptors" for specific hormones *Prohormone* - *inactive precursors that can be cleaved into one or more active hormones*

recognize the signs and proper treatment of complications from insulin therapy

*Hypoglycemia is the most common* side effect of insulin treatment -15 every 15 -Give 15 grams of glucose and recheck Blood glucose every 15 -If patient is found in a coma or unresponsive the use of a glucagon kit may be required Lipodystrophy can also occur at injection site due to insulins damage of the adipose tissue -Rotate sites of injection frequently to avoid complications

Differentiate the major causes hypocalcemia

*Hypoparathyroidism* -Post surgical/post parathyroidectomy -Autoimmune -Congenital -Familial *Infiltrative disease:* -Hemochromatosis -Amyloidosis -Wilson's disease Also Hypomagnesemia Hungry bone syndrome Genetic disorders Vitamin D deficiency Vitamin D resistance PTH resistance Acute pancreatitis Rhabdomyolysis Severe illness Blood transfusion -citrate chelates ca++ Drugs

List the main hormones of the hypothalamus, anterior and posterior pituitary, their target organs, and their primary function.

*Hypothalamus:* - Corticotropin-releasing hormone (CRH) = Stimulates the pituitary release of ACTH - Gonadotropin-releasing hormone (GnRH) = Stimulates the pituitary to release LH and FSH - Thyotropin-releasing hormone (TRH) = Stimulates the pituitary to release thyroid-stimulating hormone (TSH) - Growth hormone releasing hormone (GHRH) = Stimulates the release of growth hormone (GH) from the pituitary - Somatostatin = Inhibits the release of GH from the pituitary - Dopamine = Inhibits the release of prolactin from the pituitary *Anterior Pituitary:* - Growth hormone (GH) = Promotes the body's growth and development - Prolactin (PRL) = Controls milk production - Adrenocorticotropic hormone (ACTH) = Stimulates the adrenal cortex - Thyroid-stimulating hormone (TSH) = Stimulates the thyroid hormone - Luteinizing hormone (LH) = Stimulates the production of sex hormones - Follicle-stimulating hormone (FSH) = stimulates follicle development (women) or sperm production (men) *Posterior Pituitary* - Vasopressin (AVP) (AKA Antidiuretic Hormone (ADH)) = Helps control the body's water and electrolyte levels - Oxytocin = Promotes uterine contraction during labor and activates milk ejection in nursing woman

Treatment of symptomatic Hypercalcemia or Hypercalcemia above 14 mg/Dl

*IV hydration* (let them pee out the calcium) Saline 200-300 ml/hr Adjust to keep urine output 100-150mL/h *Calcitonins* (causes rapid decrease) *Bisphosphonates*( longer effects, 24-36 hours) Calcimimetics (cinacalcet (inhibit PTH release, not your first choice because it won't strengthen the bone like the first two, indicated in severe non surgical primary, or secondary HPTH with CKD) Dialysis in (some cases) Glucocorticoids when indicated *Identify/Treat underlying cause* Diuresis with lasix *only if signs of fluid overload*

List the various classes of anti hyperglycemic medications, give an example and describe the MOA, indications, contraindications, and common side effects of each class

*Insulin MOA*: -Binds to alpha subunit -simulates tyrosine kinase activity at beta subunit (inside cell) -activates glucose transporter to surface -allows glucose entry into cell -Insulin causes skeletal muscle and liver absorption of glucose & fat cells absorption of lipids (stored as triglycerides) *4 types of Insulin*: *Rapid acting*: -Aspart or lispro (novolog or humolog) -onset in 15 minutes -last 3-5 hours *Short acting*: -Regular insulin -Onset 30-60 min -Last 5-8 hours *Intermediate acting*: -NPH (humulin N) -onset 30-60 min -lasts 5-8 hours *Long acting*: -Glargine (lantus) -onset 90 min -lasts 24 hrs at steady amount *Common side effects*: -Hypoglycemia *Common uses:* (these are used in type 1 diabetics or poorly controlled type 2 diabetics) Insulin pump: rapid acting insulin only insulin injections: 1 shot of Lantus daily and then a rapid acting insulin for meal bolus and hyperglycemia corrections

Define the following disease: Pathophysiology, Epidemiology, Risk factors, Clinical presentation, physical findings, diagnostic evaluation, Ddx, management for *Type 2 Diabetes*

*What:* -The insulin receptors have become desensitized to insulin. -Beta cells become less sensitive to glucose and decrease insulin secretion. -Insulin-stimulated muscle glycogen synthesis is the major culprit -FFA levels rise but insulin release is not stimulated -There fore blood glucose levels rise! *Who:* 95% of diabetics are Type II (30 Mil + another 8 undiagnosed) *Risk factors:* -OBESITY (this is the greatest risk factor) -Genetics (family history) -Diet/ exercise level -History of gestational diabetes for women *Presentation:* -Hyperglycemia (fasting BG > 126 or HbA1c above 6.5%) -polyuria -polydypsia -Will often see dyslipidemia, HTN, and atherosclerosis at the same time -(Often won't come in until there has been a complication with an undiagnosed TIIDM i.e. MI, Stroke, neuropathy, retinopathy etc) -candida infections -Blurred vision -Fatigue *Diagnosis:* -HbA1c above 6.5% -Fasting plasma glucose of > 126 mg/dL DDX: Type 1 diabetes *Management:* -Diet/ lifestyle can make huge improvements BUT -Metformin is the first-line therapy (start 500 mg at night, then if tolerated go to 500mg BID) -If patient does not get A1c between 6.5- 7% then you start adding treatment anti hyperglycemic drugs insulin preparations etc.

Define the following disease: Pathophysiology, Epidemiology, Risk factors, Clinical presentation, physical findings, diagnostic evaluation, Ddx, management for *Type 1 Diabetes*

*What:* Autoimmune disease in which the immune system destroys Beta Islet cells in the pancreas (no more insulin production). -1/3 of the risk is due to genes (HLA-DR3 or HLA-DR4) 95% of T1DM have these genes -2/3 is due to an environmental trigger maybe a virus, maybe cow milk...? *Why:* Typically there is an environmental insult that kicks this off usually a infection (cocksackie virus) and then the immune system over reacts *Who:* 5% of all diabetics are Type I (3 mil affected) and its onset is usually youth but can happen at any age *Risk factors:* Family history of diabetes or autoimmune disorders (i.e. hypothyroidism) *Presentation:* young kid following an infection -Hyperglycemic -Fruity breath -THIRSTY -PEEING A LOT -Agitated -WEIGHT LOSS *-DKA (Yikes)* *Diagnostic evaluation:* -Blood glucose above 200 and Symptoms -Fasting blood glucose above 126 *DDX?* -Type II diabetes in obese or older Pt -C-peptide will guide when in doubt (low in type 1, high in type 2) *Treatment*-aka good luck -*Life long insulin therapy* -Insulin shots + Blood glucose monitoring -*Check HgA1c every 3 months (goal is below 7%)* -Screen for microalbuminemia yearly -ACE-I/ ARB if protein evident -Check BUN/ Creatinine yearly -Check eyes yearly by Optho (diabetic retinopathy) -Check feet every visit (ulcers/ Diabetic neuropathy) -Check LDL (statin when above 100) -40 Y.O Bday gift= statin -vaccinate against pneumococcal and everything else

Given a clinical scenario be able to recognize and treat *hypoparathyroid tetany*

*emergency* *involuntary muscle contractions* Sx of *acute hypocalcemia* due to hypoparathyroidism Tx: 1) Ensure airway is clear 2) *Intravenous Calcium gluconate* 10-20 ml of 10% solution given *slowly* until tetany ceases To prevent recurrence, prescribe calcium supplements and vitamin D orally https://www.youtube.com/watch?v=zbfe_m2djQM (Skip to 15 second mark)

*Insulin*

*lowers blood glucose*, Produced by the *beta cells in islet of langerhans* T1/2 of 5-8 minutes only sugar lowering hormone in the body. Controlled directly by* blood glucose levels, glucogon*, and indirectly by GH, glucocorticoids, and thyroid hormone Primarily targets the organs responsible for energy storage *liver, muscles, adipose* The insulin receptor on a cell is *tyrosine kinase*, Once it is activated it induces *Glut-4 transporters* to *allow glucose into adipose and muscle* Inhibits breakdown of stored proteins, lipids, and glycogen for energey, *(decreases blood amino acids)* Increases fat synthesis, which lowers glycerol which can be used for gluconeogenesis and *lowers fatty acids and keto acids in the blood*

4. Summarize the lifelong healthcare needs unique to transgender patients

- *every year for DM regardless of age* -Beginning at age 20 screen routinely for *lipids* -HEPATIC function, screen annually LFTs - PROLACT: measure serum prolactin levals at baseline, 12 months following ignition of treatment and biannually after *OSTEOPOROSIS* - transgender female: screen if fracture risk is elevated or if they have stopped taking hormone therapy. - transgender male 10 years after initiation of testosterone therapy and then every 10 years after that. - all pt that have undergone *gonadectomy started on vitamins D and calcium supplements* (especially for testosterone therapy) *PROSTATE CANCER:* -MtF: exam not PsA- will be falsely low bc of estrogen *Rctal/transvaginal exam at 50* *BEAST CANCER* MTF- at higher risk bc of prolongued exposure to estrogen

Define the following diseases/disorders to include the pathophysiology, epidemiology, risk factors (if any), clinical presentation, physical findings, diagnostic evaluation, differential diagnoses, and management plan: • *Pheochromocytoma*

- Chromaffin tumors that usually arise in the adrenal medulla. Can be extra-adrenal, along the sympathetic chain in the abdomen, thorax, or neck, and rarely in sympathetic tissue in the walls of the urinary bladder Genes associated with pheochromocytoma: MEN 2A, MEN 2B, VHL, NF-1, PGL genes Clinical presentation: Patients by not have all of these symptoms - *refractory hypertension* (because the tumors secrete NE) - *Triad: headache/palpitations/sweating* - Skin blanching also anxiety, nausea, fatigue, weight loss, abdominal and chest pain Diagnosis: - Elevated Plasma metanephrines - 24 hour urine free catecholamines (dopamine, epinephrine, norepinephrine) Plasma catcholamines are NOT reliable! Drugs can interfere with the assay (Methyldopa, Tricyclic antidepressants, Adrenergic blockers) Treatment = *Surgical excision of tumor* - Preoperative Medical treatment (Should be instituted as soon as the diagnosis is made) - *α - adrenergic blockers (Phenoxybenzamine)* - ß-blockers (to control tachyarrhythmias)

Discuss the following medical complications to include treatment options associated with diabetes: *Disorders of the Skin*

-Acanthosis Nigricans (darkening and thickening of skin) commonly seen in T2DM (pictured) -bacterial infections -Diabetic dermopathy AKA shin spots (light brown/ red spots on shins or body) -digital sclerosis (hardening of skin on hands) -Fungal infections -You name it diabetics probably have it

2. Describe the underlying neuroscientific and genetic basis of the transgender patient

-Biologic theory supports the concept of sexual differentiation in the brain and relies on the notion that the brain is dismorphic and in utero develops into male or female. Zygotic twins higher concordance for homosexuality in MZ vs. DZ twins Suggests genetic role in development of gender dysphoria - environmental factors involved in rearing not well studied -Human brain mosaic- Challenged sexually dimorphic view of human brains (i.e., ♀ or ♂ brain) ---> Conclusion: human brains exhibit sex differences but do not belong to one of 2 distinct categories: male brain/female brain Neither biological nor psychological studies satisfactorily explain gender identify disorder/gender dysphoria NO ONE KNOWSSSS??

When given a clinical scenario demonstrate the ability to choose the appropriate therapy/ medication for a patient presenting with hyperglycemic condition or crisis

-Check K level this will determine whether you can give insulin first or if you need to correct K level prior to insulin treatment. -If K is > 3.3 then give insulin -If K is < 3.3 then replace serum K prior to insulin administration (KCL) -Regular insulin or rapid acting insulin is given as bolus followed by basal rate until normal blood glucose is reached. -Saline

Discuss the following medical complications to include treatment options associated with diabetes: *Neuropathy* Both peripheral and autonomic

-Diabetic Neuropathy's are the most Common complication of Diabetes affecting 50% of T2DM -Risk of nerve damage increases with time as diabetic and poor glycemic control *Peripheral Neuropathy* -Most common -Stocking-glove pattern (affects longest nerves of the body) -Presents as bilateral and distal neuropathy -Sensory first then motor -Pain, numbness, dulled perceptions, weakness, etc. -Can lead to ulcers, trauma, fractures -Loss of proprioception (fall risk) *Screening* -Check every office visit in diabetic Pt -Check for light touch, reflexes, vibratory sensations *Treatment*-Difficult -Manage blood glucose is most important -Nortriptyline/ Desipramine (tricyclic anti-depressants) are useful for pain -Phenytoin/ Gabapentin/ SSRI/ anticonvulsants can help with pain -Nerve stimulation/ Nerve block may help -Capsaicin cream *Autonomic Neuropathy* -Occurs due to amount of time with the disease -ANS problems (blood pressure, pulse, *gastrointestinal activity*, bladder function, and *erectile dysfunction*.) Gastroparesis-Metoclopramide or erythromycin Diarrhea- Loperamide ED-give Sildenafil or Tadalafil (Pde-I)

List and describe the current criteria/ guidelines used for the diagnosis of diabetes

-Fasting plasma glucose is commonly used to diagnose type 2 diabetics (above 126 mg/dL is diabetic) -Oral Glucose tolerance test (less common) give 75 g glucose then measure Blood glucose 2hrs later normal < 140 abnormal >200 mg/dL -A1c is used for monitoring control and guiding treatment A1c above 6.5% is diabetic

Discuss the following medical complications to include treatment options associated with diabetes: *Nephropathy*

-Most common cause of End stage renal disease (ESRD) -As kidneys become damaged you see glomerular hypertrophy, renal enlargement, expansion of the mesangial matrix, thickening of the GBM, all resulting in *glomerulosclerosis* -Causes microalbuminemia (small amount of albumin in urine) -This is a risk factor for CVD Diagnosis: - 30 mg/d< Microalbuminemia >300 mg/d -Albuminemia >300 mg/d -measured twice within 3-6 months -Decline in GFR *Prevention:* -Control HTN!!!! (ACE-I) -Control blood glucose *Screening:* -Start screening yearly for T2DM at diagnosis -Start screening yearly 5 years after diagnosis in T1DM

what is the *Somogyi Effect*

-Overnight the patient goes low (hypoglycemia) and then the body responds by releasing epinephrine and glucagon. -The next morning the patient is high (hyperglycemia) -The initial low in the night is usually due to going too long without eating or eating a meal before bed and overcorrecting with insulin

Define the following diseases/disorders to include the pathophysiology, epidemiology, risk factors (if any), clinical presentation, physical findings, diagnostic evaluation, differential diagnoses, and management plan: • *Adrenal insufficiency*

2 types: 1) Primary adrenal insufficiency (Addison disease): dysfunction or absence of the adrenal cortices. 2) Secondary adrenal insufficiency: deficient secretion of ACTH from the pituitary. - Iatrogenic adrenal insufficiency: Caused by chronic exogenous steroid use and withdrawal. Symptoms: Can be vague and none specific symptoms - Weakness / Fatigue - Anorexia (Ask how much are you eating?) - Nausea / Vomiting - Abdominal pain - Diarrhea - Muscle or joint pain Physical Exam: - *Weight loss* - Sparse axillary hair - *Hypotension* - *Hyperpigmentation, especially in creases, pressure areas, and nipples (Primary only)* Labs: - Low AM cortisol - Low serum sodium - High serum potassium, calcium and BUN *Major concern = Adrenal Crisis* Medical emergency, can be life threatening Treat: IV Fluids, IV Steroids Diagnosis: 1st = *AM cortisol* (Cortisol levels should peak between 6:00-8:00 AM) > 19 = intact hypothalamic-pituitary-adrenal (HPA) axis < 5 = definite adrenal insufficiency 6 -18 = indeterminate (Do Cosyntropin (ACTH) Stimulation test) 2nd = *Cosyntropin (ACTH) Stimulation test =* - Cosyntropin is a synthetic bioactive form of ACTH - Inject and measure serum cortisol at 0, 30, and 60 minutes - A cortisol level > 19 at any point indicates that the adrenal glands can be stimulated adequately and does not have Primary Adrenal Insufficiency (Addison's disease) Treatment: *Treat the underlying cause*

Case #3 - 60 year old man with HTN, HLD, rheumatoid arthritis evaluated in ICU for hypotension due to sepsis - On vasopressors with persistent hypotension - Medications: Metoprolol, ASA, Lipitor - Vitals: Bp= 90/55, P= 104 - Labs: Na= 130, K= 5.0, Bicarb= 24, albumin= 4.0g/dL, Hcrt= 30 - Random cortisol: 3 ug/dL Appropriate management? A. Stress dose hydrocortisone B. Measure 8AM cortisol, ACTH C. Fluids, antibiotics, and supportive care D. Stimulation test

A. Stress dose hydrocortisone

Define the following diseases/disorders to include the pathophysiology, epidemiology, risk factors (if any), clinical presentation, physical findings, diagnostic evaluation, differential diagnoses, and management plan: • *Primary (hyper)aldosteronism* Conns Syndrome

AKA Conn's syndrome Excessive production of aldosterone by the adrenal glands independent of any regulation by the RAS system. This increases the activity of the Na/K pumps in the cortical collecting tubules causing: - Sodium retention = ECF volume expansion and HTN - Potassium loss = hypokalemia - Increases the secretion of H+ ions = metabolic alkalosis Causes: - Adrenal adenoma (aldosterone producing adenoma) - Adrenal hyperplasia (bilateral) - Familial Hyperaldosteronism - Adrenal carcinoma Clinical features: - HTN - Headache - Fatigue /weakness - Polydipsia - nocturnal polyuria - absence of peripheral edema Labs: - Hypokalemia - Metabolic Alkalosis Screening tests: - 8 am plasma aldosterone (PA) and plasma renin (PRA) levels. Obtain PA/PRA ratio. Plasma Aldosterone > 15 ng/dL Ratio (Ald/Renin) > 20 (4 -10 is normal) Confirmatatory testing: Saline or salt loading test with 24-hour urine aldosterone (If salt fails to suppress aldosterone = Hyperaldosteronism) Diagnosis - Serum Aldosterone > 15 ng/dL - *Plasma aldosterone to renin ratio > 20* = further evaluatation -Saline or salt loading with 24-hour urine aldosterone > 12 mcg/24 hrs (urine aldosterone + high urine sodium confirms diagnosis) To locate cause: - Adrenal CT then, if needed - Adrenal venous sampling (AVS) for aldosterone levels (This is to identify which adrenal is FOR SURE the culprit for the hyperaldosteronism) Treatment ○ Adenoma = surgical resection ○ Bilateral hyperplasia = spironolactone (surgery is NOT indicated)

Define the following diseases/disorders to include the pathophysiology, epidemiology, risk factors (if any), clinical presentation, physical findings, diagnostic evaluation, differential diagnoses, and management plan: • *Cushing's syndrome*

AKA Hypercortisolism Pathophysiology: *increased production of glucocorticoids* (cortisol is principal glucocorticoid). Causes - *Iatrogenic = due to overuse of steroids (most common)* - Cushing's disease (over secretion of ACTH from the anterior pituitary) - Adrenal adenomas and carcinomas - Ectopic ACTH production (abnormal ACTH secretion from somewhere in the body) Clinical presentation: - *Central obesity* - *Wasting of arms and legs* - rounding of face (moon face) - dorsocervical fat pad (buffalo hump) - proximal muscle weakness - cognitive dysfunction - mood swings - severe fatigue - *hirsutism* Physical findings: - *purple/red striae* - *thinning of skin on hands* - Hyper pigmentation of the nail beds (ectopic cause) - inability to stand up from a squat or out of a car seat Diagnosis - Midnight salivary cortisol value - *24 hour urine free cortisol* - *1 mg overnight dexamethasone suppression test* = 1st - 1 mg Dexamethasone is taken at 11PM 2nd - 8 AM serum cortisol level is drawn the next morning (plasma cortisol value less than 1.8 mg/dL is normal) To identify and locate the cause: 1st: *Check plasma ACTH level* - Low / Suppressed (<10) = adrenal source - Normal or high (20-100) = Cushing's disease - Very high ACTH (>100) = ectopic source 2nd: Locate the tumor / source - CT scan for adrenal or pituitary cause suspected - PET scan for Ectopic causes Treatment: Based on the cause: - Iatrogenic = tapering of glucocorticoid - Pituitary (Cushing Disease) = surgery or transsphenoidal ablation of pituitary adenoma - Adrenal adenoma or carcinoma = surgery (adrenalectomy)

Define the following diseases/disorders to include the pathophysiology, epidemiology, risk factors (if any), clinical presentation, physical findings, diagnostic evaluation, differential diagnoses, and management plan. *Paget disease*

Absnormal osteoclast activity causing an increase in bone turnover and remodeling with unkown cause Often Asymptomatic, "deep" Bone pain is 1st Sx, worse at night Sx may include deformities, arthritis,Fx, Kyphosis, bowed tibias, large head, deafness Dx: X ray abnormalities, bone leasions, deformities and changes in bone density *alkaline phosphatase* is elevated, Increased uptake on bone scan Tx: only treat if symptomatic or with skull involvment 1st line- *Bisphosphonates* (prevent loss of bone mass) 2nd line-Calcitonin

Define the following diseases/disorders to include the pathophysiology, epidemiology, risk factors (if any), clinical presentation, physical findings, diagnostic evaluation, differential diagnoses, and management plan. • Acromegaly

Acromegaly literally means "extremity enlargement" - Overgrowth of bone in acral (peripheral) areas *Caused by GH hypersecretion in adults* (long bone epiphyses are fused). GH stimulates the release of IGF-1 for liver and other tissue *Almost always caused by a GH secreting pituitary adenoma* Clinical features: Acral enlargement - Widening "doughy" hands and feet (Carpal tunnel very common) - Coarsening of facial features (Older photographs can be helpful) - Deep coarse voice Hypogonadism - Sleep apnea - Glucose intolerance Labs: - High serum IGF-1 (5 X nml) - High GH levels - High PRL (Secreted by the tumor) - Confirmed by Oral Glucose Tolerance Test (GH will not be suppressed) Imaging: MRI = pituitary tumor Treatment: - *Pituitary transsphenoidal microsurgery* (removal of the adenoma while preserving anterior pituitary function) If pituitary does not return to normal, use medication: - Cabergoline: Dopamine agonist - Octreotide: Somatostatin analog - Pegvisomant: GH receptor blocker

How do you treat hypocalcemia

Acute symptomatic- *IV calcium gluconate* Cardiac monitoring Chronic aysmptomatic Oral calcium Oral activated vitamin D (1,25 oh) (*calcitriol*)

Dual-energy x-ray absorptiometry (DXA) evaluation of bone density, and levels of osteoperosis

Bone density scan Focus on the *T score* Cannot differentiate beteen osteoperosis and osteomalacia

Thyroid hormones (T3,T4)

Gland in the front of the throat that produces *Thyroxine (T4) and triiodothyronine (T3)* that are iodiniated derivitives of tyrosine* A tyrosine with one iodide is MIT, and a tyrosine with two iodinds is DIT. DIT+DIT= T4 (halflife 7 days) DIT+MIT=T3 (halflife 1 day) Both require binding proteins for transport Normally you produce 90% T4 and 10% T3, but if you are defecient you make 50/50 Thyroid takes in Iodide with a Na cotransporter "iodide trapping" against its gradient Both hormones are "thyroid hormone" but T3 is the active one, T4 needs to be converted to T3 by the kidneys before it can cause calorigenesis *Increases metabolism of body tissues* (heats you up, burns off your fat) and the metabolism of what you eat Vital in early CNS development Similar effect of GH with bone growth. Teeth, skin, hair, nerves, heart, and GI all need thyroid hormone to grow *Hyperthyroidism* Increased appetite, weight loss, heat intolerance, increased sweating (graves disease- antibodies mimic TSH and you excrete too much T3/T4 and get a goiter and decreased TSH) *Hypothyroidism* Reduced appatite, weight gain, cold intolerance, decreased sweating. (Hashimotos- Antibodies inhibit t3/t4, get a goiter and decreased TSH) *iodide deficiency* Cant make t3/t4 get goiter and increased TSH

3. Discuss the clinical diagnosis, evaluation and multi-disciplinary treatment of the transgender patient.

Goal: "Lasting personal comfort with the gendered self to maximize overall psychological well being and self fulfillment." Multidisciplinary approach to therapy: pediatrician; internist (endocrinologist); psychiatrist; behavioral therapist; gynecologist; urologist Sequential 3-part evaluation and therapy *STEP1*: *Mental health evaluation* may take 1 yr to determine risk/benefit to pt and document gender dysphoria exists Psychiatric comorbidities identified and treated "minority stress"; depression; anxiety; PTSD *"Real-life" test: ≥ 1 yr living full-time as person of desired sex* *STEP 2:* *Hormone therapy* Must complete diagnostic/psychotherapeutic phase 1st Goals: 1) suppress sex characteristics associated with patient's natal sex; 2) induce characteristics of the desired sex F--> m -testosterone therapy M --> F - estrogen therapy (estradiol) -androgen suppressing therapy *STEP 3:* *Sex reassignment surgery* Procedures are irreversible Fertility may be problematic due to hormonal and/or surgical route MTF: sperm count can recuperate after hormone free-period FTM: oocyte or embryo cryopreservation prior to hormonal therapy FTM: some can achieve successful pregnancies after testosterone withdrawal *Feminizing or masculinizing genital surgery requires referrals from 2 mental health professionals (WPATH) + 1 yr hormone therapy and 1 continuous yr of living in desired gender role*

Outline the feedback mechanisms of common endocrine systems *HPG*

How your body releases sex hormones 1) Hypothalamus releases gonadotropin releasing hormone *GnRH* in regular bursts 2) GnRH stimulates release of FSH and LH from anterior pituitary. *Men* (Makin sperm) 3) LH stimulates certain cells in testes (leydig cells) to release testosterone. 4)Testosterone and FSH regulate sertoli cells in sperm maturation. (sertoli are sperm babysitters) 5) Sertoli cells secrete inhibin (negative feedback on FSH) and Leydig cells (sertoli a little bit) produce activin which stimulates FSH (positive feedback) 6) (negative feedback) Testosterone inhibits hypothalamus and pituitary via Estrogen (a biproduct of testosterone) and Dihidrotestosterone *Female* (Releasing Eggs) 3) During the menstrual cycle LH and FSH stimulate the ovarian follicle (with egg) to produce estradiol. 4)Most of the time LH and FSH are negative feedback and *prevent* the hypothalamus from releasing GnRH, HOWEVER, just before ovulation, estradiol causes a surge in LH which turns it into a positive feedback and *leads* to ovulation, forming corpus luteum, and progesterone release. 5. After ovulation, LH promotes progesterone and estradiol production by corpus luteum, which show negative feedback on GnRH and LH. 6) Progesterone is negative feedback for LH and FSH release

Define the following disease: Pathophysiology, Epidemiology, Risk factors, Clinical presentation, physical findings, diagnostic evaluation, Ddx, management for *Hypoglycemia*

Hypoglycemia aka Low blood sugar problems but not related to type I diabetes insulin overdose. Hypoglycemia symptoms: -*Sweating* -Blurred or double vision -confusion, erratic behavior -*Sense of impending doom* -*Fatigue* -Lethargy -Anger/ hostility -I like to describe it as Weak. *Whipples triad:* (if patient has these 3 then they have hypoglycemia) 1. Symptoms of low glucose 2. Measured blood glucose at time when symptoms are present (BG < 40 mg/dL) 3. Symptoms are improved with administration of glucose : ) Two types: *Fasting Hypoglycemia* -imbalance between hepatic glucose production and peripheral glucose utilization -(Too much glucose gets pulled out of the blood during times of fasting) -Sx will be during the night, in between meals or after exercise -Causes insulinoma, insulin, adrenal insufficiency *Non-Fasting Hypoglycemia* -postprandial (negative 72 hour fast)? -Commonly seen post gastric surgery *Treatment:* -Give sugar -15 every 15 (give 15 grams sugar then recheck in 15 minutes) -A parenteral glucagon emergency kit (1 mg) should be provided to every patient with diabetes who is receiving insulin therapy.

Define the following disease: Pathophysiology, Epidemiology, Risk factors, Clinical presentation, physical findings, diagnostic evaluation, Ddx, management for *Hyperglycemic Hyperosmolar state (HHS)*

Like DKA but you will see HIGH blood sugar (like around 800-1000) -Greater degree of dehydration -NO KEYTONES -NO metabolic acidosis -Serum osmolality > 320 mmol/L -Lactic acids are in greater quantity (poor tissue profusion) -More common in TYPE II diabetics or those who were previously undiagnosed type II. Treatment: -Give insulin -Give fluids -run usual labs

what is the *Dawn Phenomenon*

Overnight the body releases Growth hormone and then cortisol in the early morning. These hormones counteract the activity of insulin. so when a patient wakes up they experience a high blood sugar due to the hormone effects.

Define the following diseases/disorders to include the pathophysiology, epidemiology, risk factors (if any), clinical presentation, physical findings, diagnostic evaluation, differential diagnoses, and management plan. • Hypopituitarism

Pathophysiology: diminished secretion of one or more pituitary hormones, results from: - anterior pituitary gland destruction or - dysfunction of hypothalamic stimulatory-inhibitory factors that normally regulate pituitary function Etiology: - Pituitary tumors - Surgery - Pituitary apoplexy - Sheehan's syndrome (Infarction of the anterior pituitary gland during childbirth) - Pituitary infiltrative diseases (Sarcoidosis, Hemosiderosis, Tuberculosis) General characteristics: - all or some of the hormones released from the anterior pituitary may be absent. Usually LH/FSH/GH are lost first, then TSH, and finally ACTH Clinical presentation: *Depends on which hormones are lost* - Reduced GH = growth failure, increased LDL, increased risk of heart disease - Reduced prolactin = failure to lactate (Sheehan's syndrome) - Reduced ACTH = adrenal insufficiency - Reduced TSH = hypothyroidism - Reduced gonadotropins = infertility, amenorrhea, loss of secondary sex characteristics, diminished libido - Reduced ADH = diabetes insipidus - Reduced melanocyte-stimulating hormone = decreased skin/hair pigmentation - Pituitary Mass = Headache, Visual field disturbances (Bilateral temporal hemianopsia) Diagnosis: - Clinical picture - Evaluation of target organ function - *Measurement of pituitary hormone levels and of their target hormones level* - *Stimulation tests to confirm deficiencies* - MRI of the hypothalamus and pituitary when a mass lesion is suspected Treatment: - GIVE THEM WHAT THEY ARE MISSING - Surgery if indicated - Lifetime hormone replacement

Define the following diseases/disorders to include the pathophysiology, epidemiology, risk factors (if any), clinical presentation, physical findings, diagnostic evaluation, differential diagnoses, and management plan. • Syndrome of Inappropriate Antidiuretic Hormone Secretion (SIADH)

Pathophysiology: excess ADH is secreted from posterior pituitary or an ectopic source leading to water retention and excretion of concentrated urine. Two major effects: - hyponatremia - volume expansion *No edema because natriuresis occurs despite hyponatremia* Causes: - neoplasms - CNS disorders - pulmonary disorders - positive pressure ventilators - drugs - postoperative state Clinical features - Acute hyponatremia (lethargy, weakness → seizures, coma, death) - Chronic hyponatremia - Urine osm > serum osm - Euvolemia Diagnosis: - diagnosis of exclusion of all other causes of hyponatremia Treatment: - Correct underlying cause - *Water restriction* - Hypertonic saline - ADH receptor antagonists (Tolvalptan & Conivaptan)

Define the following diseases/disorders to include the pathophysiology, epidemiology, risk factors (if any), clinical presentation, physical findings, diagnostic evaluation, differential diagnoses, and management plan: • *Addison's disease*

Primary Adrenal insufficiency Causes: - *Autoimmune (most common cause)* • Spontaneous • PGA syndrome (Polyglandular Autoimmune) - Infiltration/destruction of adrenal • TB • Amyloidosis • Hemochromotosis • Bilateral adrenal metastases - Hemorrhage (pts on anitcoagulation (Hepiran)) - Waterhouse-Friderichsen syndrome (adrenal hemorrhage due to meningococcemia) Signs and Symptoms: SEE PREVIOUS CARD Labs: - Low AM serum cortisol - High serum ACTH (> 200 pg/mL) - Neutropenia, lymphocytosis & easinophilia - Low serum sodium - High serum potassium, calcium and BUN Diagnosis: 1st = *AM Serum cortisol* (<19) 2nd (If needed) = Cosyntropin Stimulation test - *Cortisol that never raises above 19 is diagnostic of Addison's* Treatment: Glucocorticoid (Cortisol) replacement therapy - Hydrocotisone - Cortisone Acetate - Prednisone Mineralocorticoid (Aldosterone) replacement therapy - Fludrocortisone acetate (retains sodium)

Androgens

Principle is *testosterone*, second is DHT Primarily made in * testes* , Makes males grow into male patterns, increases muscle mass (protein anabolic activity) Functions change in *three* stages: Fetus- development of internal and external male genitalia Puberty- Growth, growth in genitalia, pubes, muscles ETC Adult- Maintains masculinity, libido, sexual potency, and regulates sperm production. Drops off a little bit in old age, but not as much as estrogen in women women get a little bit in the adrenal gland, and the ovaries

Describe the pathophysiology, epidemiology, risk factors, clinical presentation, diagnostic evaluation, and management plan of: *Gynecomastia*

Result of an imbalance between estrogen and androgen action at the breast tissue level WHAT: -"Man boobs" -*Benign proliferation of the glandular tissue of the male breast* -*Increased ratio of estrogen: testosterone* CAUSES: -Drugs (spironolactone) -Persistent pubertal gynecomastia -Idiopathic -Cirrhosis or malnutrition -Primary hypogonadism -Hyperthyroidism -Germ cell tumors testes (Inc HCG -Develops in about 50% of athletes who abuse androgens and anabolic steroids -Common in teenagers who are very tall or overweight. CLINICAL PRESENTATION: -*Excess glandular tissue when compared to normal* -Must be distinguished from carcinoma or mastitis DIAGNOSIS: -*Usually done on physical exam (see picture)* -Palpable enlargement of the male breast, often asymmetric or unilateral. -*Glandular gynecomastia characterized by tenderness* -*Fatty gynecomastia typically nontender* -Must be distinguished from carcinoma or mastitis. -Investigation of unclear cases should include a chest radiograph to search for bronchogenic or metastatic carcinoma TREATMENT: -*Observation* -Pre pubertal-tends to resolve spontaneously -If medication related; *stop agent if possible, will regress spontaneously* -*Give testosterone in DEFICIENT patients only* -Selective estrogen receptor modulator (SERM) = *Tamoxifen or Raloxifene* (decreases breast volume) -Aromatase inhibitors (not in adolescents) -*Surgical correction* reserved for patients with persistent or severe gynecomastia

How is thyroid hormone produced? (Not an objective, just physiology stuff)

See picture

More in depth look at the hormones of the Hypothalamus

These include the major inhibitory or releasing hormones that will act on the pituitary *Corticotropin releasing hormone* (CRH), which is part of the hormone system regulating carbohydrate, protein, and fat metabolism as well as sodium and water balance in the body *Gonadotropin releasing hormone* (GnRH), which helps control sexual and reproductive functions, including pregnancy and lactation (i.e., milk production) *Thyrotropin releasing hormone* (TRH), which is part of the hormone system controlling the metabolic processes of all cells and which contributes to the hormonal regulation of lactation *Growth hormone releasing hormone* (GHRH), which is an essential component of the system promoting the organism's growth *tells pituitary to release growth hormone* *Somatostatin* which also affects bone and muscle growth but has the *opposite effect as that of GHRH *inhibits pituitary from releaseing growth hormone* GH stimulates its release in negative feedback *Dopamine* a substance that functions primarily as a neurotransmitter but also has some hormonal effects, such as inhibiting release of prolactin and repressing lactation until it is needed after childbirth.

Diagnosing Hyperparathyroidism

They have elevated serum calcium, Normal limit is 10.4 Measure the *PTH*. If its elevated then they probably have PHPT. Normal is 11-54 Measure *urine calcium* to confirm. If it is above 250 mg/dl then they have PHPT. IF is is lower then they might have Famelial hypocaliuric hypercalcemia (not an objective)

Define the following disease: Pathophysiology, Epidemiology, Risk factors, Clinical presentation, physical findings, diagnostic evaluation, Ddx, management for *Insulin Resistance Syndrome* AKA metabolic syndrome

This is not type II diabetes yet but if you are here you will likely develop other bad things. *Perfect storm for developing TIIDM and CVD:* -Genetic predisposition -lack of exercise -body fat distribution in the abdomen -Obese -Hyperlipidemia -Hyperinsulinemia -Hyperglycemia *Dx: Requires 3/5* -Abdominal waist circumference in men ≥ 40in in women ≥35 in -Serum triglycerides ≥150 mg/dL or drug treatment for elevated triglycerides -HDL <40 mg/dL in men and <50 mg/dL in women or drug treatment for low HDL cholesterol -Blood pressure ≥130/85 mmHg or drug treatment for elevated blood pressure -*Fasting plasma glucose (FPG) ≥100 mg/dL or drug treatment for elevated blood glucose* -A1c above 5.7% *Tx:* Start to control risk factors -You can give them metoformin or thiazolidinediones (actos) -Take statins -Take ACE-I -Lose weight (10% will improve a lot) -Exercise -Improve diet

Describe the pathophysiology, epidemiology, risk factors, clinical presentation, diagnostic evaluation, and management of: *Hypothyroidism* There are many types! -(Other cards will hit these in more detail)

WHAT IS IT?: *Thyroid fails to secrete adequate amount of hormone* -Hypothyroidism is the most common thyroid disorder -Hypothyroidism may be associated with menstrual disorders, infertility, risk of miscarriage, and other complications of pregnancy DUE TO A VARIETY OF CAUSES: *Primary hypothyroidism* ● Autoimmune: Hashimoto thyroiditis ● Iatrogenic: Thyroidectomy ● Drug induced: Iodine deficiency, iodine excess, lithium, amiodarone, antithyroid drugs ● Congenital: thyroid agenesis, thyroid dysgenesis, hypoplastic thyroid, biosynthetic defect *Secondary hypothyroidism* ● Hypothalamic dysfunction: Neoplasms, TB, sarcoidosis, langerhans cell histiocytosis, hemochromatosis, radiation treatment ● Pituitary dysfunction: Neoplasms, pituitary surgery, postpartum pituitary necrosis, idiopathic hypopituitarism, Cushing's syndrome, radiation treatment PRESENTATION (SEE PICTURE): -Children: learning disabilities, mental retardation, short stature, delayed bone age, delayed puberty -Adults: *fatigue, cold intolerance, weight gain*, constipation, menstrual irregularities, *thin brittle nails, thin and coarse hair, dry/coarse/cold skin*, periorbital edema, delayed reflexes, bradycardia, LABS: -*Elevated TSH* -*Low levels of T4 (thyroxine)* -This is because the anterior pituitary produces more TSH in an effort to stimulate the thyroid gland into producing more thyroid hormone. TREATMENT: -Daily use of the synthetic thyroid hormone *T4-Levothyroxine (Levothroid, Synthroid, others)* which restores adequate hormone levels

Describe the pathophysiology, epidemiology, risk factors, clinical presentation, diagnostic evaluation, and management plan of: *Polycystic ovarian syndrome* -Dont go by the name for this one

WHAT: *A Spectrum of a Disease* -accounts for 20% of *secondary* amenorrhea cases -*Most common endocrine abnormality in reproductive aged women* -Onset prior to age 40 -Exact cause is unknown, but obesity and high serum insulin levels contribute to the syndrome CLINICAL PRESENTATION: -*Irregular periods* (This is the most common characteristic) -*Excess facial and body hair (hirsutism)* -*Adult acne or severe adolescent acne* -*Male-pattern baldness (androgenic alopecia)* DIAGNOSIS: -Diagnosis of exclusion! -Blood tests show *elevated serum testosterone or free testosterone level* and *signs of androgen excess* -*Ultrasound shows polycystic ovaries (enlarged and contain numerous small fluid-filled sacs which surround the eggs)* -*Low FSH, LH, and low estradiol* *WE USE THE ROTTERDAM CRITERIA TO DIAGNOSE* (Must have 2/3 for diagnoses) -Elevated testosterone or S+S of Hyperandrogenism -Irregular periods -Ultrasound shows polycystic ovaries TREATMENT: -*Weight loss (lifestyle change)* -*Oral contraceptives with combined estrogen and progestin* (HRT helps with bone health) -Clomiphene/Metformin to help ovulation

Describe *Amiodarone induced hypothyroidism*

WHAT: *Amiodarone is a widely used antiarrhythmic drug that is 37% iodine by weight* -Can cause both hyper and *hypothyroidism* (more common) DIAGNOSIS: -*Use of Amiodarone in the past* -*Labs show increased TSH, decreased free T4 and free T3* (for the purpose of this test, HYPO lab values) TREATMENT: -*Stop amiodarone if possible* -*Give synthroid* (aim to keep free T4 at high normal) -Do NOT try to normalize TSH as this requires high dose and clearly causes hyperthyroidism -Patients with clear-cut type 2 amiodarone-induced thyrotoxicosis are usually also treated with prednisone

Describe the pathophysiology, epidemiology, risk factors, clinical presentation, diagnostic evaluation, and management of: *Postpartum thyroiditis*

WHAT: *Autoimmune thyroiditis that occurs soon after delivery in 7.2% of women* -Pay attention on the test if the woman has recently given birth! WHY: -The autoimmunity may be triggered by the accumulation of fetal cells in the maternal thyroid during pregnancy, a condition known as microchimerism -*The condition appears to be very similar to Hashimoto's disease* -70% chance of recurrence after subsequent pregnancies -Occurs in women who have high levels of thyroid peroxidase antibody in the first trimester of pregnancy or immediately after delivery PRESENTATION: (Can be Triphasic) -*Patient MAY initially present with symptoms similar to those of an overactive thyroid (see hyperthyroidism card)* --->*This typically occurs 2-6 weeks postpartum and lasts 2-3 months.* -*Hypothyroid phase then occurs (so, their signs and symptoms would change) --->These symptoms occur 4-8 months after and can last 9-12 months.* -*Thyroid function will return to normal in most women eventually* -KNOW that it is possible for women to develop ONLY hypothyroidism and not the "hyper phase". DIAGNOSIS: -Physical exam -*Blood tests show high TSH and low T3 and T4 (in hypo phase)* -*Over 80% have antithyroid antibodies. Thyroglobulin (Tg Ab)/Thyroid peroxidase (TPO Ab)* TREATMENT: -*Most women who develop postpartum thyroiditis don't need treatment during the hyperthyroid or hypothyroid phases* -MONITOR THE THYROID FUNCTION IF SEVERE, TREAT BASED ON SYMPTOMS -*Beta blocker (propranolol) during HYPER phase* -*Thyroid hormone therapy for six to 12 months levothyroxine (Levothroid, Synthroid, others) during if HYPO symptoms develop* -Ipodate sodium or iopanoic acid?

Describe the pathophysiology, epidemiology, risk factors, clinical presentation, diagnostic evaluation, and management of: *Thyroid storm*

WHAT: *Critical complication of poorly treated/untreated hyperthyroidism (thyrotoxicosis)* -Can be precipitated by surgery, radioactive iodine therapy or severe stress (uncontrolled DM, MI, acute infection). -Rare but has high mortality rate. It used to be 25% now with earlier diagnosis it is treatable, mortality is much lower SIGNS AND SYMPTOMS: -*Delirium* -*Severe tachycardia/AFib* -Vomiting -Diarrhea -*Dehydration* -*High fever* TREATMENT: -*Admit to ICU* -*IV fluids/cooling measures* -Monitor for lytes/BP and temp -*Hydrocortisone IV every 8 hours for 48 hours* (then start to decrease 50% every day till off) -Steroids (to decrease t4-t3 conversion) -*Methimazole (MMI)* or propylthiouracil (PTU) -*Start beta blocker*

Describe the pathophysiology, epidemiology, risk factors, clinical presentation, diagnostic evaluation, and management of: *Graves' disease*

WHAT: *Graves' disease is an immune system disorder that results in the overproduction of thyroid hormones (hyperthyroidism)* -More common in women (Male/Female =1/7) -Peaks between 20-40 years of age PATHOPHYS: -The immune system mistakenly produces the antibody TSI (thyroid stimulating immunoglobulin). TSI mimics thyroid-stimulating hormone (TSH), which is the hormone that signals your thyroid to produce more T3 and T4. -The eye also has TSH receptors so patients with Graves are at increased risk for eye involvement CLINICAL PRESENTATION: -*Signs and symptoms of hyperthyroidism* (see previous card) -*Goiter (often with bruit)* -Clubbing of fingers -Skin thickening (least common) PLUS GRAVES' OPPHTHALMOPATHY (PICTURE) -*Exophthalmos/proptosis* (bulging eyes) -*Stare and lid lag* -Vision problems DIAGNOSIS: -Physical exam -*Blood sample shows low TSH and high T4 and/or T3* -*High TSI levels*, the key antibody in pt's with Graves! -Radioactive iodine uptake (helps determine if Graves' disease or another condition is the cause of the hyperthyroidism) -Ultrasound (useful in people who can't undergo radioactive iodine uptake, such as pregnant women) -*MRI or CT of the orbit* shows orbital muscle enlargement TREATMENT: -*Antithyroid drugs* (propylthiouracil or methimazole) -*Radioactive iodine therapy (131i, RAI)* -*Beta-blockers (Propranolol)* -Surgery (either subtotal or total thyroidectomy)

Describe the pathophysiology, epidemiology, risk factors, clinical presentation, diagnostic evaluation, and management of: *Toxic mutinodular goiter*

WHAT: *Hyperfunctioning thyroid nodules that produce hyperthyroidism are known as toxic multinodular goiter (also called Plummer disease)* -*Common causes of hyperthyroidism, second in prevalence only to Graves' disease* -More common in the elderly and in iodine deficient regions. -About 95% of thyroid nodules are benign (5% are cancer) -REFER this patient for further testing PRESENTATION: -*Patient will have symptoms of hyperthyroidism* (see card) -Cardiovascular complications occur commonly in elderly patients, and a history of atrial fibrillation, congestive heart failure, or angina may be present. -*Classic clinical picture: a hyperthyroid patient with a palpable thyroid nodule that corresponds to an area of increased radioiodine concentration on thyroid scan.* DIAGNOSIS: -*Nodular goiter on physical exam* -*Suppressed TSH, elevated T3 and T4* -*Radioactive Iodine scan (I 123 scan) shows a "hot nodule" - see picture here* TREATMENT: -*Surgery is the definitive treatment for patients with large toxic nodular goiter* (total or near-total thyroidectomy) -*Radioiodine (RAI, 131I) ablation therapy* may be used to treat patients with toxic nodular goiter (usually in after surgery has been done) *RAI should not be given to women with a toxic multinodular goiter within 3 months prior to a planned conception*

Describe the pathophysiology, epidemiology, risk factors, clinical presentation, diagnostic evaluation, and management of: *Hyperthyroidism (Thyrotoxicosis)*

WHAT: *Hyperthyroidism (overactive thyroid) is a condition in which your thyroid gland produces too much of the hormone thyroxine.* -Hyperthyroidism can accelerate your body's metabolism -Prevalence in the general population 1.2% (more common in women) -*Graves' Disease is the most common form* SIGNS/SYMPTOMS: -*Sudden weight loss* -*Rapid heartbeat (tachycardia)* -*Atrial fibrillation may be the presenting manifestation of hyperthyroidism* -Increased appetite -*Nervousness, anxiety and irritability* -Tremor -*Increased sweating* -*Exophtalmos if Graves' disease* -*Increased sensitivity to heat* -Changes in bowel patterns -An enlarged thyroid gland (goiter)* -Fatigue, muscle weakness -*Skin thinning* -*Fine, brittle hair* -Hyperglycemia DIAGNOSIS: -*Medical history and physical exam* -*Blood tests show high levels of thyroxine (T4) and low/nonexistent amounts of TSH* -Radio iodine uptake test: Determines how much iodine the thyroid gland has absorbed (high uptake of radioiodine indicates the thyroid gland is producing too much thyroxine) -*Thyroid scan*: a radioactive isotope injected into a vein of the elbow/hand. A camera then produces an image of the thyroid gland on a computer screen while the patient is lying down. TREATMENT: -*Radioactive iodine* (causes the gland to shrink and symptoms to subside) -Anti-thyroid medications such as propylthiouracil (PTU) and *methimazole (Tapazole)*. -*Beta blockers* (for heart rate/prevent palpitations) -*Surgery (thyroidectomy)* in bad cases; pt will need lifelong treatment with levothyroxine (Levoxyl, Synthroid, others) to supply the body with normal amounts of thyroid hormone afterwards NOTE: Check CBC's and LFT's prior to starting medical treatment

Describe the pathophysiology, epidemiology, risk factors, clinical presentation, diagnostic evaluation, and management plan of: *Klinefelter's syndrome*

WHAT: *Klinefelter syndrome is a genetic condition that results when a boy is born with an extra copy of the X chromosome.* *The most common chromosomal abnormality among males* -Incidence of about 1:500 -Often isn't diagnosed until adulthood. -*It is caused by the expression of an abnormal karyotype, classically 47,XXY* -Up to 75% of affected boys experience some gynecomastia at puberty -Common cause of PRIMARY hypogonadism PRESENTATION: -*Testes in adolescence become firm, fibrotic, small*, and nontender to palpation. -*Behavior can be aggressive* -Tall stature -*Small phallus (Micropenis)* - Think Connor... -Infertility -*Sparse body hair* -Low sperm count -*Gynecomastia* DIAGNOSIS: -*Serum testosterone is usually low and gonadotropins (FSH/LH) are elevated* -*Chromosome (karyotype) analysis to confirm diagnosis* -Rule out other conditions before treatment TREATMENT: -No direct cure -*Testosterone replacement therapy* --->200 mg q 2 weeks -Breast tissue removal -Counseling -Monitoring (Testosterone should be between 400-700)

Describe the pathophysiology, epidemiology, risk factors, clinical presentation, diagnostic evaluation, and management plan of: *Menopause*

WHAT: *No menses for > 1 year* -*Happens to women between the ages of 45-55* WHY: -Ovarian and endocrine changes ultimately result in the *depletion of oocyte stores and the cessation of ovarian estrogen production* CLINICAL PRESENTATION: -*Vasomotor symptoms (hot flushes) are experienced by 60-80% of women at menopause* -*Vaginal atrophy/dryness* -Sleep disturbances -*Mood swings* -Reduced libido TREATMENT: -If they have a uterus = *estrogen + progesterone* -*No uterus = *estrogen* -Can consider use of an SSRI

Describe the pathophysiology, epidemiology, risk factors, clinical presentation, diagnostic evaluation, and management plan of: *Pseudogynecomastia*

WHAT: -*Build up of fat behind, around, and under nipples!* -The look and feel of gynecomastia is *not as a result of excess breast tissue but fat accumulation* -Common among elderly men, particularly when there is associated weight gain TREATMENT: -*Exercise* -Surgery if needed

Describe the pathophysiology, epidemiology, risk factors, clinical presentation, diagnostic evaluation, and management plan of: *HYPOgonadotrophic (secondary) hypogonadism*

WHAT: -*Decrease in one or both of the two major functions of the testes: sperm production or testosterone production.* -*In SECONDARY hypogonadism, these abnormalities result from disease of the hypothalamus or pituitary.* CAUSES: -Kallmann syndrome -Idiopathic (no known cause) -*Pituitary disorders (Tumors: Prolactinoma, Adenoma)* -Inflamatory disorders -HIV/AIDS -*Medications/narcotics/supplements(GNC stuff)/ recreational drugs* -Obesity CLINICAL PRESENTATION: -*Signs and symptoms depend on when the condition develops* *In Fetal development:* -Impaired growth of the external sex organs -Female genitals/Ambiguous genitals -Underdeveloped male genitals *In Puberty:* -*Delay puberty* -Decreased development of muscle mass -*Lack of deepening of the voice* -Impaired growth of body hair -Impaired growth of the penis and testicles -*Development of breast tissue (gynecomastia)* *In Adulthood:* -*Erectile dysfunction* -Infertility -Decrease in beard and body hair growth -*Decrease in muscle mass* -Development of breast tissue (gynecomastia) -Mental and emotional changes. -Fatigue -*Decreased sex drive* -Hot flashes DIAGNOSIS: -Labs show: *Serum testosterone concentration and/or the sperm count are below normal* (LOW) --->*Need 2 levels for diagnosis* (CHECK IN THE AM) -Labs show: *Serum gonadotropin (LH and FSH) levels are decreased or normal* -Check SHBG by equilibrium dialysis -Semen analysis -Pituitary imaging -Genetic studies TREATMENT: -*Pituitary hormones* may stimulate sperm production and restore fertility -*Testosterone replacement therapy can be used IF FERTILITY IS NOT AN ISSUE* (pt can loose fertility) --->200 mg q 2 weeks (Testosterone should be between 400-700) -*A pituitary tumor may require surgical removal*, medication, radiation or the replacement of other hormones

Describe the pathophysiology, epidemiology, risk factors, clinical presentation, diagnostic evaluation, and management plan of: *HYPERgonadotrophic (primary) hypogonadism*

WHAT: -*Decrease in one/both of the two major functions of the testes: sperm production or testosterone production.* -*In PRIMARY hypogonadism these abnormalities result from problem with the testes* CAUSES: -*Testicular injury* -*Klinefelter syndrome* -Undescended testicles -Hemochromatosis -Cancer treatment CLINICAL PRESENTATION: -*Signs and symptoms depend on when the condition develops* *In Fetal development* -Impaired growth of the external sex organs -Female genitals/Ambiguous genitals -Underdeveloped male genitals *In Puberty* -*Delay puberty* -Decreased development of muscle mass -*Lack of deepening of the voice* -Impaired growth of body hair -Impaired growth of the penis and testicles -*Development of breast tissue (gynecomastia)* *In Adulthood* -*Erectile dysfunction* -Infertility -Decrease in beard and body hair growth -Decrease in muscle mass -Development of breast tissue (gynecomastia) -Mental and emotional changes. -*Fatigue* -Decreased sex drive -Hot flashes DIAGNOSIS: -Physical exam to note sexual development (is the pt's pubic hair, muscle mass and size of testes consistent with pt's age?) -Labs show: *Serum testosterone concentration and/or the sperm count are below normal* (Low) --->*Need 2 levels for diagnosis (CHECK IN THE AM)* -Labs show: *Serum LH and/or FSH concentrations above normal/increased* (High) -Check SHBG by equilibrium dialysis -Semen analysis -Pituitary imaging -Genetic studies TREATMENT: -Depends on the cause and whether pt is concerned about fertility -*Hormone replacement (testosterone replacement therapy, or TRT)* -*Testosterone replacement therapy* --->200 mg q 2 weeks (Testosterone should be between 400-700) -Assisted reproduction

Describe the pathophysiology, epidemiology, risk factors, clinical presentation, diagnostic evaluation, and management plan of: *Turner's syndrome*

WHAT: -Turner syndrome (TS) is a condition in which *a female is partly or completely missing an X chromosome* -*Pts are infertile* ASSOCIATED WITH: -*Coarctation of the aorta (10%)* -*Renal abnormalities (50%) - Horseshoe Kidney* -Hearing abnormalities -Short 4th metacarpals -Inflammatory bowel disease -*Aortic Valuvlar Disease* -*Hashimoto's thyroiditis* CLINICAL PRESENTATION: -micrognathia (small chin/jaw) -low set ears -fishlike mouth -*Short stature* -*Shield-like, square chest* -*Neck is short, broad, and webbed* DIAGNOSIS: -*Diagnosis is based on physical signs and genetic testing* -Obvious on physical exam -Gonadal dysgenesis - ovaries replaced with fibrous tissue (seen in CT scan) TREATMENT: -No cure for Turner syndrome is known -*Estrogen and cyclic progesterone replacement* at puberty will result in normal pubertal development -*Growth hormone* sometimes used to gain max height

Describe the pathophysiology, epidemiology, risk factors, clinical presentation, diagnostic evaluation, and management of: *Subacute (DeQuervain) thyroiditis*

WHAT: Also called de Quervain thyroiditis, granulomatous thyroiditis, and giant cell thyroiditis—is relatively common WHY: -*It is believed to be caused by a viral infection and often follows an upper respiratory tract infection.* -young and middle-aged women are most commonly affected. PRESENTATION: -*Viral upper respiratory infection* (KNOW THIS!) -*Enlarged and extremely painful thyroid* -*Signs/symptoms of hyperthyroidism (initially)* -Fever DIAGNOSIS: -*Physical exam* (History of viral infection!) -*Increased WBC, ESR (erythrocyte sedimentation rate), and C-reactive protein.* -Labs will fluctuate with state of disease. It is "triphasic" and go from hyperthyroidism --> hypothyroidism ---> euthyroidism TREATMENT: -*Beta blocker (Propranolol)* during the hyperthyroid phase -Pain can usually be managed with *NSAIDS and corticosteroids*-Ipodate sodium or iopanoic acid (corrects elevated T3 levels and is continued until the serum T4 level normalizes) -Levothyroxine if hypothyroidism develops

How testosterone, DHT, and estrogen are made and role of DHT, and the chart showing negative feedback in testosterone production

cholesterol (side chain clevage enzyme) proegnenolone (3BHSD) progesterone (17 alpha-hydroxylase and 17,20 desmolase) androstenedione (type IIIB- HSD) *Testosterone* If you have aromatase you can make estrogen, if you have 5 Alpha reductase then you can make DHT dihydrotestosterone ( converting enzymes)

Pituitary (aka the hypophysis... if you hear that don't get confused)

located in the brain directly below the hypothalamus consists of two parts: *the anterior pituitary which is it's own organ, and the posterior pituitary which is part of the brain* The hypothalamus tells it what to do

Hormone feedback mechanisms.

many hormones produced by target glands are regulated by pituitary hormones, which in turn are controlled by hypothalamic hormones. HPA-Hypothalamic pituitary *adrenal* HPG-Hypothalamic pituitary *gonadal* HPT-Hypothalamic pituitary *thyroidal*


Kaugnay na mga set ng pag-aaral

Civilization of Asia: Japan (Final)

View Set

Lesson 113 - GFCI, AFCI, and Other Special-Purpose Receptacles Quiz

View Set

Consumer Behavior Exam 2 Jeopardy

View Set

Biology Game Questions: Biochemistry

View Set

Diabetes Mellitus (Ch. 48-Section 10)

View Set

Final Study Guide for "Projekt 1065."

View Set

N144 AQ Pregnancy, labor, childbirth, postpartum

View Set